Show that $(sqrt{y^2-x}-x)(sqrt{x^2+y}-y)=y iff x+y=0$












25












$begingroup$



Let $x,y$ be real numbers such that
$$left(sqrt{y^{2} - x,,}, - xright)left(sqrt{x^{2} + y,,}, - yright)=y$$
Show that $x+y=0$.




My try:
Let
$$sqrt{y^2-x}-x=a,sqrt{x^2+y}-y=bLongrightarrow ab=y$$
and then
$$begin{cases}
y^2=a^2+(2a+1)x+x^2cdotscdots (1)\
x^2=b^2+(2b-1)y+y^2cdotscdots
end{cases}$$
$(1)+(2)$
then
$$x=-dfrac{a^2+b^2+(2b-1)ab}{2a+1}cdotscdots (3)$$
so
$$x+y=ab-dfrac{a^2+b^2+(2b-1)ab}{2a+1}=dfrac{(a-b)(2ab-a+b)}{2a+1}$$
we take $(3)$ in $(2)$,we have
$$b^2+(2b-1)y+y^2-x^2=dfrac{(2ab-a+b)(2a^3b+a^3+3a^2b-2ab^3+ab^2+4ab-b^3+b)}{(2a+1)^2}=0$$



so
$$(2ab-a+b)=0$$
or
$$2a^3b+a^3+3a^2b-2ab^3+ab^2+4ab-b^3+b=0$$
if
$$2ab-a+b=0Longrightarrow x+y=dfrac{(a-b)(2ab-a+b)}{2a+1}=0$$
and if
$$2a^3b+a^3+3a^2b-2ab^3+ab^2+4ab-b^3+b=0$$
I don't prove
$$x+y=dfrac{(a-b)(2ab-a+b)}{2a+1}=0?$$










share|cite|improve this question











$endgroup$












  • $begingroup$
    We can't apply inequalities here, as WA shows in (wolframalpha.com/input/…) together with (wolframalpha.com/input/…). (The variable $a$ takes a negative value as well as a positive one, so we don't have $LHSle RHS$ or $RHSle LHS$ consistently)
    $endgroup$
    – chubakueno
    Oct 29 '13 at 2:55












  • $begingroup$
    Yes,But This is different problem,Thank you
    $endgroup$
    – china math
    Oct 29 '13 at 4:37










  • $begingroup$
    Maybe it helps to call $f(x,y) = x+y$, leave all that mess in terms of $f(x,y), x$ and $y$, and check that $partial f/partial x = partial f / partial y = 0$, and see that $f(x,y) = 0 $ at least once.
    $endgroup$
    – Ivo Terek
    Jul 15 '14 at 4:15










  • $begingroup$
    By conjecture, x+y=0, so x = -y. Substitute -y for x in the equation. Solve left side, y=y.
    $endgroup$
    – Paul Magnussen
    Aug 26 '14 at 17:50










  • $begingroup$
    Why isn't it correct to prove by the conjecture $y=-x$?
    $endgroup$
    – rae306
    Aug 26 '14 at 18:29
















25












$begingroup$



Let $x,y$ be real numbers such that
$$left(sqrt{y^{2} - x,,}, - xright)left(sqrt{x^{2} + y,,}, - yright)=y$$
Show that $x+y=0$.




My try:
Let
$$sqrt{y^2-x}-x=a,sqrt{x^2+y}-y=bLongrightarrow ab=y$$
and then
$$begin{cases}
y^2=a^2+(2a+1)x+x^2cdotscdots (1)\
x^2=b^2+(2b-1)y+y^2cdotscdots
end{cases}$$
$(1)+(2)$
then
$$x=-dfrac{a^2+b^2+(2b-1)ab}{2a+1}cdotscdots (3)$$
so
$$x+y=ab-dfrac{a^2+b^2+(2b-1)ab}{2a+1}=dfrac{(a-b)(2ab-a+b)}{2a+1}$$
we take $(3)$ in $(2)$,we have
$$b^2+(2b-1)y+y^2-x^2=dfrac{(2ab-a+b)(2a^3b+a^3+3a^2b-2ab^3+ab^2+4ab-b^3+b)}{(2a+1)^2}=0$$



so
$$(2ab-a+b)=0$$
or
$$2a^3b+a^3+3a^2b-2ab^3+ab^2+4ab-b^3+b=0$$
if
$$2ab-a+b=0Longrightarrow x+y=dfrac{(a-b)(2ab-a+b)}{2a+1}=0$$
and if
$$2a^3b+a^3+3a^2b-2ab^3+ab^2+4ab-b^3+b=0$$
I don't prove
$$x+y=dfrac{(a-b)(2ab-a+b)}{2a+1}=0?$$










share|cite|improve this question











$endgroup$












  • $begingroup$
    We can't apply inequalities here, as WA shows in (wolframalpha.com/input/…) together with (wolframalpha.com/input/…). (The variable $a$ takes a negative value as well as a positive one, so we don't have $LHSle RHS$ or $RHSle LHS$ consistently)
    $endgroup$
    – chubakueno
    Oct 29 '13 at 2:55












  • $begingroup$
    Yes,But This is different problem,Thank you
    $endgroup$
    – china math
    Oct 29 '13 at 4:37










  • $begingroup$
    Maybe it helps to call $f(x,y) = x+y$, leave all that mess in terms of $f(x,y), x$ and $y$, and check that $partial f/partial x = partial f / partial y = 0$, and see that $f(x,y) = 0 $ at least once.
    $endgroup$
    – Ivo Terek
    Jul 15 '14 at 4:15










  • $begingroup$
    By conjecture, x+y=0, so x = -y. Substitute -y for x in the equation. Solve left side, y=y.
    $endgroup$
    – Paul Magnussen
    Aug 26 '14 at 17:50










  • $begingroup$
    Why isn't it correct to prove by the conjecture $y=-x$?
    $endgroup$
    – rae306
    Aug 26 '14 at 18:29














25












25








25


10



$begingroup$



Let $x,y$ be real numbers such that
$$left(sqrt{y^{2} - x,,}, - xright)left(sqrt{x^{2} + y,,}, - yright)=y$$
Show that $x+y=0$.




My try:
Let
$$sqrt{y^2-x}-x=a,sqrt{x^2+y}-y=bLongrightarrow ab=y$$
and then
$$begin{cases}
y^2=a^2+(2a+1)x+x^2cdotscdots (1)\
x^2=b^2+(2b-1)y+y^2cdotscdots
end{cases}$$
$(1)+(2)$
then
$$x=-dfrac{a^2+b^2+(2b-1)ab}{2a+1}cdotscdots (3)$$
so
$$x+y=ab-dfrac{a^2+b^2+(2b-1)ab}{2a+1}=dfrac{(a-b)(2ab-a+b)}{2a+1}$$
we take $(3)$ in $(2)$,we have
$$b^2+(2b-1)y+y^2-x^2=dfrac{(2ab-a+b)(2a^3b+a^3+3a^2b-2ab^3+ab^2+4ab-b^3+b)}{(2a+1)^2}=0$$



so
$$(2ab-a+b)=0$$
or
$$2a^3b+a^3+3a^2b-2ab^3+ab^2+4ab-b^3+b=0$$
if
$$2ab-a+b=0Longrightarrow x+y=dfrac{(a-b)(2ab-a+b)}{2a+1}=0$$
and if
$$2a^3b+a^3+3a^2b-2ab^3+ab^2+4ab-b^3+b=0$$
I don't prove
$$x+y=dfrac{(a-b)(2ab-a+b)}{2a+1}=0?$$










share|cite|improve this question











$endgroup$





Let $x,y$ be real numbers such that
$$left(sqrt{y^{2} - x,,}, - xright)left(sqrt{x^{2} + y,,}, - yright)=y$$
Show that $x+y=0$.




My try:
Let
$$sqrt{y^2-x}-x=a,sqrt{x^2+y}-y=bLongrightarrow ab=y$$
and then
$$begin{cases}
y^2=a^2+(2a+1)x+x^2cdotscdots (1)\
x^2=b^2+(2b-1)y+y^2cdotscdots
end{cases}$$
$(1)+(2)$
then
$$x=-dfrac{a^2+b^2+(2b-1)ab}{2a+1}cdotscdots (3)$$
so
$$x+y=ab-dfrac{a^2+b^2+(2b-1)ab}{2a+1}=dfrac{(a-b)(2ab-a+b)}{2a+1}$$
we take $(3)$ in $(2)$,we have
$$b^2+(2b-1)y+y^2-x^2=dfrac{(2ab-a+b)(2a^3b+a^3+3a^2b-2ab^3+ab^2+4ab-b^3+b)}{(2a+1)^2}=0$$



so
$$(2ab-a+b)=0$$
or
$$2a^3b+a^3+3a^2b-2ab^3+ab^2+4ab-b^3+b=0$$
if
$$2ab-a+b=0Longrightarrow x+y=dfrac{(a-b)(2ab-a+b)}{2a+1}=0$$
and if
$$2a^3b+a^3+3a^2b-2ab^3+ab^2+4ab-b^3+b=0$$
I don't prove
$$x+y=dfrac{(a-b)(2ab-a+b)}{2a+1}=0?$$







calculus algebra-precalculus arithmetic radicals






share|cite|improve this question















share|cite|improve this question













share|cite|improve this question




share|cite|improve this question








edited Sep 5 '17 at 2:35









George N. Missailidis

627318




627318










asked Oct 29 '13 at 2:01









china mathchina math

10.2k631117




10.2k631117












  • $begingroup$
    We can't apply inequalities here, as WA shows in (wolframalpha.com/input/…) together with (wolframalpha.com/input/…). (The variable $a$ takes a negative value as well as a positive one, so we don't have $LHSle RHS$ or $RHSle LHS$ consistently)
    $endgroup$
    – chubakueno
    Oct 29 '13 at 2:55












  • $begingroup$
    Yes,But This is different problem,Thank you
    $endgroup$
    – china math
    Oct 29 '13 at 4:37










  • $begingroup$
    Maybe it helps to call $f(x,y) = x+y$, leave all that mess in terms of $f(x,y), x$ and $y$, and check that $partial f/partial x = partial f / partial y = 0$, and see that $f(x,y) = 0 $ at least once.
    $endgroup$
    – Ivo Terek
    Jul 15 '14 at 4:15










  • $begingroup$
    By conjecture, x+y=0, so x = -y. Substitute -y for x in the equation. Solve left side, y=y.
    $endgroup$
    – Paul Magnussen
    Aug 26 '14 at 17:50










  • $begingroup$
    Why isn't it correct to prove by the conjecture $y=-x$?
    $endgroup$
    – rae306
    Aug 26 '14 at 18:29


















  • $begingroup$
    We can't apply inequalities here, as WA shows in (wolframalpha.com/input/…) together with (wolframalpha.com/input/…). (The variable $a$ takes a negative value as well as a positive one, so we don't have $LHSle RHS$ or $RHSle LHS$ consistently)
    $endgroup$
    – chubakueno
    Oct 29 '13 at 2:55












  • $begingroup$
    Yes,But This is different problem,Thank you
    $endgroup$
    – china math
    Oct 29 '13 at 4:37










  • $begingroup$
    Maybe it helps to call $f(x,y) = x+y$, leave all that mess in terms of $f(x,y), x$ and $y$, and check that $partial f/partial x = partial f / partial y = 0$, and see that $f(x,y) = 0 $ at least once.
    $endgroup$
    – Ivo Terek
    Jul 15 '14 at 4:15










  • $begingroup$
    By conjecture, x+y=0, so x = -y. Substitute -y for x in the equation. Solve left side, y=y.
    $endgroup$
    – Paul Magnussen
    Aug 26 '14 at 17:50










  • $begingroup$
    Why isn't it correct to prove by the conjecture $y=-x$?
    $endgroup$
    – rae306
    Aug 26 '14 at 18:29
















$begingroup$
We can't apply inequalities here, as WA shows in (wolframalpha.com/input/…) together with (wolframalpha.com/input/…). (The variable $a$ takes a negative value as well as a positive one, so we don't have $LHSle RHS$ or $RHSle LHS$ consistently)
$endgroup$
– chubakueno
Oct 29 '13 at 2:55






$begingroup$
We can't apply inequalities here, as WA shows in (wolframalpha.com/input/…) together with (wolframalpha.com/input/…). (The variable $a$ takes a negative value as well as a positive one, so we don't have $LHSle RHS$ or $RHSle LHS$ consistently)
$endgroup$
– chubakueno
Oct 29 '13 at 2:55














$begingroup$
Yes,But This is different problem,Thank you
$endgroup$
– china math
Oct 29 '13 at 4:37




$begingroup$
Yes,But This is different problem,Thank you
$endgroup$
– china math
Oct 29 '13 at 4:37












$begingroup$
Maybe it helps to call $f(x,y) = x+y$, leave all that mess in terms of $f(x,y), x$ and $y$, and check that $partial f/partial x = partial f / partial y = 0$, and see that $f(x,y) = 0 $ at least once.
$endgroup$
– Ivo Terek
Jul 15 '14 at 4:15




$begingroup$
Maybe it helps to call $f(x,y) = x+y$, leave all that mess in terms of $f(x,y), x$ and $y$, and check that $partial f/partial x = partial f / partial y = 0$, and see that $f(x,y) = 0 $ at least once.
$endgroup$
– Ivo Terek
Jul 15 '14 at 4:15












$begingroup$
By conjecture, x+y=0, so x = -y. Substitute -y for x in the equation. Solve left side, y=y.
$endgroup$
– Paul Magnussen
Aug 26 '14 at 17:50




$begingroup$
By conjecture, x+y=0, so x = -y. Substitute -y for x in the equation. Solve left side, y=y.
$endgroup$
– Paul Magnussen
Aug 26 '14 at 17:50












$begingroup$
Why isn't it correct to prove by the conjecture $y=-x$?
$endgroup$
– rae306
Aug 26 '14 at 18:29




$begingroup$
Why isn't it correct to prove by the conjecture $y=-x$?
$endgroup$
– rae306
Aug 26 '14 at 18:29










7 Answers
7






active

oldest

votes


















4












$begingroup$

Assume that $x<0$ and $y>0$, your statement can be written (even if we change $x$ to $-x$),




If $x,y$ are strictly positive such that $(sqrt{y^2+x}+x)(sqrt{x^2+y}-y)=y$ then $x=y$.




This equality becomes, $$overbrace{dfrac{x+sqrt{x+y^2}}{y+sqrt{y+x^2}}}^{A}=overbrace{dfrac{y}{x^2+y-y^2}}^{B}.$$ Let us start with $x>y$, clearly $B<1$. Moreover $xmapsto(x-sqrt{y+x^2})$ is increasing and $xmapsto sqrt{y+x^2}$ strictly increasing, we conclude that $xmapsto(x+sqrt{x+y^2})-(y+sqrt{y+x^2})$ is strictly increasing and thus $A>0$ when $x>y$. We deduces that $A>1$ and therefore $A$ cannot be equal to $B$.
The case $x<y$ can be treated in the same way.






share|cite|improve this answer











$endgroup$













  • $begingroup$
    @Downvoters: Would you care to explain?
    $endgroup$
    – Krokop
    Aug 26 '14 at 18:23










  • $begingroup$
    I upvoted you because I think your solution is definitely the right way to go. However, it could use more detail: we want to be more rigorous about showing that $A'(x) > 0$. Also, you should have written $A > 1$ when $x > y$.
    $endgroup$
    – heropup
    Aug 26 '14 at 21:27










  • $begingroup$
    @Krokop I think the downvoter is OP.. All of the answers in this thread have been downvoted.
    $endgroup$
    – Cameron Williams
    Aug 27 '14 at 3:09










  • $begingroup$
    @heropup Thanks for your comment, I edited to add some detail.
    $endgroup$
    – Krokop
    Aug 28 '14 at 12:07










  • $begingroup$
    @CameronWilliams It seems that he wants (always) a full detailed solution..
    $endgroup$
    – Krokop
    Aug 28 '14 at 12:12



















3












$begingroup$

Multiplication:



$y=x y-x sqrt{x^2+y}-y sqrt{-x+y^2}+sqrt{x^2+y} sqrt{-x+y^2} Rightarrow$



$x sqrt{x^2+y}+y sqrt{-x+y^2}=sqrt{x^2+y} sqrt{-x+y^2}+xy-y$



squaring both sides:



$2 x y sqrt{x^2+y} sqrt{y^2-x}+x^4+x^2 y-x y^2+y^4=
-x^3-x y+y^2-2 x y^2+2 x^2 y^2+y^3+(2 x y-2 y )sqrt{x^2+y} sqrt{-x+y^2} Rightarrow $



simplifying:



$x^4+x^3-2 x^2 y^2+x^2 y+x y^2+x y+y^4-y^3-y^2 = -2 y sqrt{x^2+y} sqrt{y^2-x}$



Squaring again:



$y^8 - 2 y^7 - 4 x^2 y^6 + 2 x y^6 - y^6 + 6 x^2 y^5 + 2 y^5 + 6 x^4 y^4 - 2 x^3 y^4 + 3 x^2 y^4 - 4 x y^4 + y^4 - 6 x^4 y^3 - 4 x^3 y^3 - 2 x y^3 - 4 x^6 y^2 - 2 x^5 y^2 + x^4 y^2 + x^2 y^2 + 2 x^6 y + 4 x^5 y + 2 x^4 y + x^8 + 2 x^7 + x^6=
-4 x^3 y^2+4 x^2 y^4-4 x y^3+4 y^5$



The only thing we have to check is how it factors as $(y+x) (p(x,y))$. Which gives us:



$(x+y)^2(x^6-2 x^5 y+2 x^5-x^4 y^2-2 x^4 y+x^4+4 x^3 y^3+2 x^3 y-x^2 y^4-4 x^2 y^3-4 x^2 y^2+2 x^2 y-2 x y^5+6 x y^4+2 x y^3+y^6-2 y^5-y^4-2 y^3+y^2)=0$






share|cite|improve this answer











$endgroup$













  • $begingroup$
    Your last inequality is wrong. Try $x=0.8$ and $y=1$.
    $endgroup$
    – Michael Rozenberg
    Nov 29 '18 at 7:07



















2












$begingroup$

In the following proof we divide the $(x,y)$ plane into regions (see diagram) and show that each region can contain no solutions except on the line $x + y = 0$.



enter image description here



Let
$$
F(x,y) = U(x,y)V(x,y) – y
$$
where
$$
U(x,y) = sqrt{y^2 – x} ,, – x
\V(x,y) = sqrt{x^2 + y} ,, – y
$$



Then solutions satisfy
$$
F(x,y) = 0
$$



Substituting $y=-x$ shows that $x+y=0$ is a solution for all x.



U is nonreal where $x>y^2$ (regions A and C in the diagram, bounded by red lines) and V is nonreal in the region $y<-x^2$ (regions B and C, also bounded by red lines). In these regions F is nonreal except possibly in region C, where U and V are both unreal. But there the condition for F to be real reduces to $x+y=0$, a subset of the known solution.



The following statements and deductions relate to the other regions of the $(x,y)$ plane, where $U$ and $V$ are real.



$U<0 Leftrightarrow x>tfrac{1}{2}(-1 + sqrt{1 + 4y^2})$ (regions I, J).



$V<0 Leftrightarrow y>tfrac{1}{2}(1 + sqrt{1 + 4x^2})$ (all regions except F, G).



$U_{x} < 0$ (regions D-K)



$V_{x} < 0 Leftrightarrow x < 0$ (regions D, E, F)



$U_{xx} < 0$ (regions D-K)



$V_{xx} < 0 Leftrightarrow y < 0$ (regions D, J, K)



where a subscript x denotes partial differentiation with respect to x.



On the diagram the lines on which $U=0$ and $V=0$ are coloured green and blue, respectively. It is easily shown that $F$ is nonzero on all the coloured lines (with sign as indicated) except at $(0,0)$ and $(1,-1)$. These lines delimit, but are excluded from, the regions A-K.



From the results above we can make the following deductions.



In region D:
$$
U>0, V>0, U_{x}<0, V_{x}<0
\F_{x} = UV_{x} + VU_{x} < 0
$$
This region is bounded on the right by the line $y<-x^2$, on which $F>0$. So $F>0$ throughout region D and it can contain no solutions.



In region E:
$$
U>0, V>0, U_{x}<0, V_{x}<0
\F_{x} < 0
$$
so here there can be no solutions other than those known to exist on the line $y=-x$.



In region F:
$$
U>0, V<0, U_{x}<0, V_{x}<0, U_{xx}<0, V_{xx}>0
\F_{xx} = UV_{xx} + VU_{xx} + 2U_{x}V_{x} > 0
$$
This region is bounded on the left by the line $V=0$ and on the right by the line $x = 0$, and on both these lines $F<0$. So the positive second derivative $F_{xx}$ means there can be no solutions $F=0$ in this region.



In a similar way, solutions can be ruled out for the following regions:



In region G, bounded on right by the line $V=0$ on which $F<0$:
$$
U>0, V<0, U_{x}<0, V_{x}>0
\F_{x}>0
$$



In region I, bounded on left by the line $U=0$ on which $F<0$:
$$
U<0, V>0, U_{x}<0, V_{x}>0
\F_{x} < 0
$$



In region J, containing a segment of the known solution line $x+y=0$ on which $F=0$:
$$
U<0, V>0, U_{x}<0, V_{x}>0
\F_{x} < 0
$$



In region K, bounded on the left and right by lines on which $F>0$:
$$
U>0, V>0, U_{x}<0, V_{x}>0, U_{xx}<0, V_{xx}<0
\F_{xx} = UV_{xx} + VU_{xx} + 2U_{x}V_{x} < 0
$$



Finally, in region H:
$$
U>0, V>0, U_{x}<0, V_{x}>0
$$
and we note that $U_{x}<0$ in region G also, so for a given $y$,
$U<U_{max}$, where $U_{max} = U(0,y) = y$



For the same value of y, $V<V_{max}$, where $V_{max} = V(X,y)$, and X is the value of x on the right-hand boundary of the region. On this boundary, $y=sqrt{X^2+X}$, so
$$
V_{max} = V(X,y) = sqrt{X^2+y} , – y
< sqrt{X^2+X+y} , - y = sqrt{y^2 + y} ,, – y < tfrac{1}{2}.
$$



Therefore
$$
F = UV – y < U_{max} V_{max} – y < y tfrac{1}{2} – y = , –tfrac{1}{2} y < 0
$$
which completes the proof that there are no solutions other than $x+y = 0$.






share|cite|improve this answer











$endgroup$













  • $begingroup$
    Are you sure that this is a solution?
    $endgroup$
    – Michael Rozenberg
    Nov 29 '18 at 7:33





















2












$begingroup$

Assuming continuity in the area interval $ (0 < x < 1 )$ and $ (0 > y > -1 ) $ would create problems as $x$ and $y$ are not always real in these areas.



Nay, union of inside parabola areas of $ y_1 = - x^2 $ and $ y_2 = sqrt{x} $ would violate $ x + y = 0, $ which is only the common chord of intersection of $ y_1,y_2$. So the shown line joining $(0,0)$ to $(1,-1)$ does not exist as real.
E.g., $(frac12, -frac12)$ does not lie on the
common line.



enter image description here






share|cite|improve this answer











$endgroup$





















    1












    $begingroup$

    This is not a solution, but brute force can be used to remove the radicals. Let $A=y^2-x$ and $B=x^2+y$. We have



    $$sqrt{AB}-ysqrt{A}-xsqrt{B}+xy=y$$



    Isolating $sqrt{AB}$ and squaring both sides:



    $$sqrt{AB}=ysqrt{A}+xsqrt{B}+y(1-x)quad(1)$$
    $$AB=y^2A+x^2B+y^2(1-x)^2+2xysqrt{AB}+2y^2(1-x)sqrt{A}+2xy(1-x)sqrt{B}$$



    (1) allows us to remove $sqrt{AB}$. We do this and also recall what $A$ and $B$ equal.



    $$(y^2-x)(x^2+y)=y^2(y^2-x)+x^2(x^2+y)+y^2(1-x)^2+2xyleft(ysqrt{A}+xsqrt{B}+y(1-x)right)+2y^2(1-x)sqrt{A}+2xy(1-x)sqrt{B}$$



    Group $sqrt{A}$ and $sqrt{B}$ terms, then rearrange a bit:



    $$(y^2-x)(x^2+y)=y^2(y^2-x)+x^2(x^2+y)+y^2(1-x)^2+2xy^2(1-x)+2y^2sqrt{A}+2xysqrt{B}$$
    $$x^2y^2+y^3-x^3-xy=y^4-xy^2+x^4+x^2y+y^2-2xy^2+x^2y^2+2xy^2-2x^2y^2+2yleft(ysqrt{A}+xsqrt{B}right)$$
    $$y^3-x^3-xy=y^4-xy^2+x^4+x^2y+y^2-2x^2y^2+2yleft(ysqrt{A}+xsqrt{B}right)$$



    (1) allows us to sub out the quantity in parentheses:



    $$y^3-x^3-xy=y^4-xy^2+x^4+x^2y+y^2-2x^2y^2+2yleft(y(x-1)+sqrt{AB}right)$$
    $$y^3-x^3-xy=y^4-xy^2+x^4+x^2y+y^2-2x^2y^2+2y^2(x-1)+2ysqrt{AB}$$
    $$y^3-x^3-xy=y^4+xy^2+x^4+x^2y-y^2-2x^2y^2+2ysqrt{AB}$$
    $$y^3-x^3-xy-y^4-xy^2-x^4-x^2y+y^2+2x^2y^2=2ysqrt{AB}$$



    Squaring both sides, we've reached a goal of no longer having radicals.



    $$(y^3-x^3-xy-y^4-xy^2-x^4-x^2y+y^2+2x^2y^2)^2=4y^2(y^2-x)(x^2+y)$$



    I had a CAS expand this, move it all to one side, and then, as expected, $(x+y)$ factors out of it (twice).



    $$(x+y)^2 p(x,y)=0$$



    where $$p(x,y)=x^6-2 x^5 y+2 x^5-x^4 y^2-2 x^4 y+x^4+4 x^3 y^3+2 x^3 y-x^2 y^4-4 x^2 y^3-4 x^2 y^2+2 x^2 y-2 x y^5+6 x y^4+2 x y^3+y^6-2 y^5-y^4-2 y^3+y^2$$



    is a monster. It would be sufficient to show that $p(x,y)$ is never $0$ in the region of the plane where both $sqrt{A}$ and $sqrt{B}$ are defined aside from points along $x+y=0$ (like $(0,0)$). This is a pretty messy polynomial, but at least it's a polynomial.





    EDIT: This approach seems to be useless; a CAS plot of the zero set of $p$ has several components, all of which are in the region where $sqrt{A}$ and $sqrt{B}$ are defined. They must be extraneous solutions from the squaring that was done twice.






    share|cite|improve this answer











    $endgroup$





















      0












      $begingroup$

      A more generalised approach over my earlier post. This is not intended to be an exhaustive proof but an experimental one. Constructive comments are most welcome.



      Let



      $$sqrt{y^2-x}-x=Ay^nqquad cdots (1)\
      sqrt{x^2+y}-y=frac {y^{1-n}}A qquad cdots (2)\$$
      such that the original equation $$left(sqrt{y^2-x}-xright)left(sqrt{x^2+y}-yright)=y$$
      is satisfied as required.



      From $(1)$,



      $$begin{align}
      sqrt{y^2-x}&=x+Ay^n\
      y^2-x&=x^2++2Axy^n+A^2y^{2n}\
      y^{2n}A^2+2xy^nA+(x^2-y^2+x)&=0\
      A^2+frac {2x}{y^n}A+frac{(x^2-y^2+x)}{y^{2n}}&=0qquad qquad qquad qquad cdots (3)
      end{align}$$



      From $(2)$,



      $$begin{align}
      sqrt{x^2+y}&=y+frac {y^{1-n}}A\
      Asqrt{x^2+y}&=Ay+y^{1-n}\
      A^2(x^2+y)&=A^2y^2+2Ay^{2-n}+y^{2(1-n)}\
      (x^2-y^2+y)A^2-2y^{2-n}A-y^{2(1-n)}&=0\
      A^2-frac{2y^{2-n}}{x^2-y^2+y}A-frac{y^{2(1-n)}}{x^2-y^2+y}&=0qquad qquad qquad cdots (4)
      end{align}$$



      Equating coefficients of $A^1$:
      $$begin{align}frac{2x}{y^n}&=-frac{2y^{2-n}}{x^2-y^2+y}\
      y^2&=-x(x^2-y^2+y)qquad qquad qquad qquad qquad qquad cdots (5)end{align}$$



      Equating coefficients of $A^0$:



      $$begin{align}frac{x^2-y^2+x}{y^{2n}}&=-frac{y^{2(1-n)}}{x^2-y^2+y}\
      y^2&=-(x^2-y^2+x)(x^2-y^2+y)qquad cdots (6)end{align}$$



      (5)=(6):
      $$begin{align}x(x^2-y^2+y)&=(x^2-y^2+x)(x^2-y^2+y)\
      (x^2-y^2)(x^2+y^2-y)&=0\
      (x-y)(x+y)(x^2-y^2+y)&=0\
      Rightarrow x-y=&0, x+y=0, x^2-y^2+y=0end{align}$$



      Checking by substitution into the original equation shows that only
      $$x+y=0$$
      is valid.



      This graph created on desmos.com might help illustrate the approach:



      https://www.desmos.com/calculator/qrlbgbalix






      share|cite|improve this answer











      $endgroup$













      • $begingroup$
        It's an interesting approach, but I don't buy it. Equations (3) and (4) show us that $A$ is a solution to the two indicated quadratic equations. It does not follow that they must be the same equation!
        $endgroup$
        – Bungo
        Aug 23 '14 at 6:12










      • $begingroup$
        @Bungo Thanks for your comments. $A$ is actually a parameter in this case, and defines a series of curves for each equation. However at the same value of $A$ both equations intersect at a point which satisfies the original equation. The locus of this point is the solution required. Setting both quadratic equations to be the same is essentially assigning the same value of $A$ for both equations.
        $endgroup$
        – hypergeometric
        Aug 23 '14 at 9:10










      • $begingroup$
        I agree that this is valid until you square the equations. But after squaring, each equation has a second root in addition to $A$. How do we know that the second root in (3) is the same as the second root in (4)? Sorry if I'm being dense. To take a simpler example: if $x = 1$ then for any $c$ I can express this as $x-c = 1-c$. Squaring both sides, I get $x^2 - 2xc + c^2 = 1 - 2c + c^2$, or $x^2 - 2xc - (1 - 2c) = 0$. If I carry out this procedure with two different values of $c$, I get two valid equations with $1$ as a root, but the coefficients are not the same.
        $endgroup$
        – Bungo
        Aug 23 '14 at 16:40










      • $begingroup$
        That's true. By equating coefficients we arrive at one particular solution, but there may be others, e.g. where the two quadratics are not the same but have one common root...would it be correct to say that? Any suggestions on other possible approaches?
        $endgroup$
        – hypergeometric
        Aug 23 '14 at 16:58










      • $begingroup$
        Yes, that's what I was getting at. If two quadratic equations have BOTH roots in common then their coefficients are the same, up to a common factor. (Actually that's one more issue with your approach: $x^2 + 2x + 1 = 0$ and $2x^2 + 4x + 2 = 0$ have the same roots but the coefficients are not equal.) Sorry, I don't have any suggestions at the moment - the other answer is pretty thorny and I haven't read through it yet. :-)
        $endgroup$
        – Bungo
        Aug 23 '14 at 17:03



















      0












      $begingroup$

      We need to prove that $x=y$, where
      $$left(sqrt{y^{2}+x}+xright)left(sqrt{x^{2} + y}- yright)=y$$ or



      $$left(sqrt{y^{2}+x}+xright)left(sqrt{x^{2} + y}- yright)=left(sqrt{x^{2}+y}+xright)left(sqrt{x^{2} + y}- xright)$$ or
      $$left(sqrt{y^{2}+x}+xright)left(sqrt{x^{2} + y}- yright)-left(sqrt{x^{2}+y}+xright)left(sqrt{x^{2} + y}- yright)+$$
      $$+left(sqrt{x^{2}+y}+xright)left(sqrt{x^{2} + y}- yright)-left(sqrt{x^{2}+y}+xright)left(sqrt{x^{2} + y}- xright)=0$$ or
      $$left(sqrt{y^{2}+x}-sqrt{x^2+y}right)left(sqrt{x^{2} + y}- yright)+left(sqrt{x^{2}+y}+xright)(x-y)=0,$$ which gives $x=y$ or



      $$frac{(1-x-y)left(sqrt{x^{2} + y}- yright)}{sqrt{y^{2}+x}+sqrt{x^2+y}}+sqrt{x^{2}+y}+x=0,$$ which is
      $$sqrt{x^2+y}-ysqrt{x^2+y}+sqrt{(x^2+y)(y^2+x)}+xsqrt{y^2+x}+x^2+xy+y^2=0.$$
      Now, we'll consider four cases.





      1. $xgeq0$, $ygeq 0$.


      Since $$-ysqrt{x^2+y}+sqrt{(x^2+y)(y^2+x)}=sqrt{x^2+y}left(sqrt{y^2+x}-yright)geq0,$$ we obtain $x=y=0.$





      1. $xgeq0,$ $yleq0.$


      It's obvious that this case gives $x=y=0$ again.





      1. $xleq0$, $ygeq0.$


      Since, $$sqrt{(x^2+y)(y^2+x)}+xsqrt{y^2+x}=sqrt{y^2+x}left(sqrt{x^2+y}+xright)geq0,$$ it's enough to prove that
      $$x^2+xy+y^2geq(y-1)sqrt{x^2+y},$$ which is obvious for $yleq1.$



      But for $ygeq1$ by AM-GM we obtain:
      $$(y-1)sqrt{x^2+y}leqfrac{1}{2}((y-1)^2+x^2+y)$$ and it's enough to prove that
      $$x^2+xy+y^2geqfrac{1}{2}((y-1)^2+x^2+y)$$ or
      $$require{cancel} cancel{(x+y)^2+y^2+y-1geq0.}\
      (x+y)^2+y-1geq0.$$

      We see that for $ygeq1$ the equality does not occur and in the case $y<1$ the equality occurs for
      $$x^2+xy+y^2=(y-1)sqrt{x^2+y}=0,$$ which gives $x=y=0$ again.





      1. $xleq0$ and $yleq0.$


      In this case it's enough to prove that
      $$xy+xsqrt{y^2+x}geq0$$ or
      $$xleft(y+sqrt{y^2+x}right)geq0,$$ which is obvious.



      The equality occurs for $x^2+y^2=0$ and we got $x=y=0$ again.



      Done!






      share|cite|improve this answer











      $endgroup$













      • $begingroup$
        Very nice proof (+1)!
        $endgroup$
        – Andreas
        Dec 5 '18 at 15:38










      • $begingroup$
        @Andreas Thank you and thank you for your editing. Do you see that my solution it's an unique right solution in this topic?
        $endgroup$
        – Michael Rozenberg
        Dec 5 '18 at 15:53










      • $begingroup$
        Your solution is the only valid one in here, as far as I see it, which goes without calculus. There may be other approaches with calculus, I didn't check that fully.
        $endgroup$
        – Andreas
        Dec 6 '18 at 7:23











      Your Answer





      StackExchange.ifUsing("editor", function () {
      return StackExchange.using("mathjaxEditing", function () {
      StackExchange.MarkdownEditor.creationCallbacks.add(function (editor, postfix) {
      StackExchange.mathjaxEditing.prepareWmdForMathJax(editor, postfix, [["$", "$"], ["\\(","\\)"]]);
      });
      });
      }, "mathjax-editing");

      StackExchange.ready(function() {
      var channelOptions = {
      tags: "".split(" "),
      id: "69"
      };
      initTagRenderer("".split(" "), "".split(" "), channelOptions);

      StackExchange.using("externalEditor", function() {
      // Have to fire editor after snippets, if snippets enabled
      if (StackExchange.settings.snippets.snippetsEnabled) {
      StackExchange.using("snippets", function() {
      createEditor();
      });
      }
      else {
      createEditor();
      }
      });

      function createEditor() {
      StackExchange.prepareEditor({
      heartbeatType: 'answer',
      autoActivateHeartbeat: false,
      convertImagesToLinks: true,
      noModals: true,
      showLowRepImageUploadWarning: true,
      reputationToPostImages: 10,
      bindNavPrevention: true,
      postfix: "",
      imageUploader: {
      brandingHtml: "Powered by u003ca class="icon-imgur-white" href="https://imgur.com/"u003eu003c/au003e",
      contentPolicyHtml: "User contributions licensed under u003ca href="https://creativecommons.org/licenses/by-sa/3.0/"u003ecc by-sa 3.0 with attribution requiredu003c/au003e u003ca href="https://stackoverflow.com/legal/content-policy"u003e(content policy)u003c/au003e",
      allowUrls: true
      },
      noCode: true, onDemand: true,
      discardSelector: ".discard-answer"
      ,immediatelyShowMarkdownHelp:true
      });


      }
      });














      draft saved

      draft discarded


















      StackExchange.ready(
      function () {
      StackExchange.openid.initPostLogin('.new-post-login', 'https%3a%2f%2fmath.stackexchange.com%2fquestions%2f543607%2fshow-that-sqrty2-x-x-sqrtx2y-y-y-iff-xy-0%23new-answer', 'question_page');
      }
      );

      Post as a guest















      Required, but never shown

























      7 Answers
      7






      active

      oldest

      votes








      7 Answers
      7






      active

      oldest

      votes









      active

      oldest

      votes






      active

      oldest

      votes









      4












      $begingroup$

      Assume that $x<0$ and $y>0$, your statement can be written (even if we change $x$ to $-x$),




      If $x,y$ are strictly positive such that $(sqrt{y^2+x}+x)(sqrt{x^2+y}-y)=y$ then $x=y$.




      This equality becomes, $$overbrace{dfrac{x+sqrt{x+y^2}}{y+sqrt{y+x^2}}}^{A}=overbrace{dfrac{y}{x^2+y-y^2}}^{B}.$$ Let us start with $x>y$, clearly $B<1$. Moreover $xmapsto(x-sqrt{y+x^2})$ is increasing and $xmapsto sqrt{y+x^2}$ strictly increasing, we conclude that $xmapsto(x+sqrt{x+y^2})-(y+sqrt{y+x^2})$ is strictly increasing and thus $A>0$ when $x>y$. We deduces that $A>1$ and therefore $A$ cannot be equal to $B$.
      The case $x<y$ can be treated in the same way.






      share|cite|improve this answer











      $endgroup$













      • $begingroup$
        @Downvoters: Would you care to explain?
        $endgroup$
        – Krokop
        Aug 26 '14 at 18:23










      • $begingroup$
        I upvoted you because I think your solution is definitely the right way to go. However, it could use more detail: we want to be more rigorous about showing that $A'(x) > 0$. Also, you should have written $A > 1$ when $x > y$.
        $endgroup$
        – heropup
        Aug 26 '14 at 21:27










      • $begingroup$
        @Krokop I think the downvoter is OP.. All of the answers in this thread have been downvoted.
        $endgroup$
        – Cameron Williams
        Aug 27 '14 at 3:09










      • $begingroup$
        @heropup Thanks for your comment, I edited to add some detail.
        $endgroup$
        – Krokop
        Aug 28 '14 at 12:07










      • $begingroup$
        @CameronWilliams It seems that he wants (always) a full detailed solution..
        $endgroup$
        – Krokop
        Aug 28 '14 at 12:12
















      4












      $begingroup$

      Assume that $x<0$ and $y>0$, your statement can be written (even if we change $x$ to $-x$),




      If $x,y$ are strictly positive such that $(sqrt{y^2+x}+x)(sqrt{x^2+y}-y)=y$ then $x=y$.




      This equality becomes, $$overbrace{dfrac{x+sqrt{x+y^2}}{y+sqrt{y+x^2}}}^{A}=overbrace{dfrac{y}{x^2+y-y^2}}^{B}.$$ Let us start with $x>y$, clearly $B<1$. Moreover $xmapsto(x-sqrt{y+x^2})$ is increasing and $xmapsto sqrt{y+x^2}$ strictly increasing, we conclude that $xmapsto(x+sqrt{x+y^2})-(y+sqrt{y+x^2})$ is strictly increasing and thus $A>0$ when $x>y$. We deduces that $A>1$ and therefore $A$ cannot be equal to $B$.
      The case $x<y$ can be treated in the same way.






      share|cite|improve this answer











      $endgroup$













      • $begingroup$
        @Downvoters: Would you care to explain?
        $endgroup$
        – Krokop
        Aug 26 '14 at 18:23










      • $begingroup$
        I upvoted you because I think your solution is definitely the right way to go. However, it could use more detail: we want to be more rigorous about showing that $A'(x) > 0$. Also, you should have written $A > 1$ when $x > y$.
        $endgroup$
        – heropup
        Aug 26 '14 at 21:27










      • $begingroup$
        @Krokop I think the downvoter is OP.. All of the answers in this thread have been downvoted.
        $endgroup$
        – Cameron Williams
        Aug 27 '14 at 3:09










      • $begingroup$
        @heropup Thanks for your comment, I edited to add some detail.
        $endgroup$
        – Krokop
        Aug 28 '14 at 12:07










      • $begingroup$
        @CameronWilliams It seems that he wants (always) a full detailed solution..
        $endgroup$
        – Krokop
        Aug 28 '14 at 12:12














      4












      4








      4





      $begingroup$

      Assume that $x<0$ and $y>0$, your statement can be written (even if we change $x$ to $-x$),




      If $x,y$ are strictly positive such that $(sqrt{y^2+x}+x)(sqrt{x^2+y}-y)=y$ then $x=y$.




      This equality becomes, $$overbrace{dfrac{x+sqrt{x+y^2}}{y+sqrt{y+x^2}}}^{A}=overbrace{dfrac{y}{x^2+y-y^2}}^{B}.$$ Let us start with $x>y$, clearly $B<1$. Moreover $xmapsto(x-sqrt{y+x^2})$ is increasing and $xmapsto sqrt{y+x^2}$ strictly increasing, we conclude that $xmapsto(x+sqrt{x+y^2})-(y+sqrt{y+x^2})$ is strictly increasing and thus $A>0$ when $x>y$. We deduces that $A>1$ and therefore $A$ cannot be equal to $B$.
      The case $x<y$ can be treated in the same way.






      share|cite|improve this answer











      $endgroup$



      Assume that $x<0$ and $y>0$, your statement can be written (even if we change $x$ to $-x$),




      If $x,y$ are strictly positive such that $(sqrt{y^2+x}+x)(sqrt{x^2+y}-y)=y$ then $x=y$.




      This equality becomes, $$overbrace{dfrac{x+sqrt{x+y^2}}{y+sqrt{y+x^2}}}^{A}=overbrace{dfrac{y}{x^2+y-y^2}}^{B}.$$ Let us start with $x>y$, clearly $B<1$. Moreover $xmapsto(x-sqrt{y+x^2})$ is increasing and $xmapsto sqrt{y+x^2}$ strictly increasing, we conclude that $xmapsto(x+sqrt{x+y^2})-(y+sqrt{y+x^2})$ is strictly increasing and thus $A>0$ when $x>y$. We deduces that $A>1$ and therefore $A$ cannot be equal to $B$.
      The case $x<y$ can be treated in the same way.







      share|cite|improve this answer














      share|cite|improve this answer



      share|cite|improve this answer








      edited Aug 28 '14 at 12:06

























      answered Aug 26 '14 at 18:05









      KrokopKrokop

      1,014617




      1,014617












      • $begingroup$
        @Downvoters: Would you care to explain?
        $endgroup$
        – Krokop
        Aug 26 '14 at 18:23










      • $begingroup$
        I upvoted you because I think your solution is definitely the right way to go. However, it could use more detail: we want to be more rigorous about showing that $A'(x) > 0$. Also, you should have written $A > 1$ when $x > y$.
        $endgroup$
        – heropup
        Aug 26 '14 at 21:27










      • $begingroup$
        @Krokop I think the downvoter is OP.. All of the answers in this thread have been downvoted.
        $endgroup$
        – Cameron Williams
        Aug 27 '14 at 3:09










      • $begingroup$
        @heropup Thanks for your comment, I edited to add some detail.
        $endgroup$
        – Krokop
        Aug 28 '14 at 12:07










      • $begingroup$
        @CameronWilliams It seems that he wants (always) a full detailed solution..
        $endgroup$
        – Krokop
        Aug 28 '14 at 12:12


















      • $begingroup$
        @Downvoters: Would you care to explain?
        $endgroup$
        – Krokop
        Aug 26 '14 at 18:23










      • $begingroup$
        I upvoted you because I think your solution is definitely the right way to go. However, it could use more detail: we want to be more rigorous about showing that $A'(x) > 0$. Also, you should have written $A > 1$ when $x > y$.
        $endgroup$
        – heropup
        Aug 26 '14 at 21:27










      • $begingroup$
        @Krokop I think the downvoter is OP.. All of the answers in this thread have been downvoted.
        $endgroup$
        – Cameron Williams
        Aug 27 '14 at 3:09










      • $begingroup$
        @heropup Thanks for your comment, I edited to add some detail.
        $endgroup$
        – Krokop
        Aug 28 '14 at 12:07










      • $begingroup$
        @CameronWilliams It seems that he wants (always) a full detailed solution..
        $endgroup$
        – Krokop
        Aug 28 '14 at 12:12
















      $begingroup$
      @Downvoters: Would you care to explain?
      $endgroup$
      – Krokop
      Aug 26 '14 at 18:23




      $begingroup$
      @Downvoters: Would you care to explain?
      $endgroup$
      – Krokop
      Aug 26 '14 at 18:23












      $begingroup$
      I upvoted you because I think your solution is definitely the right way to go. However, it could use more detail: we want to be more rigorous about showing that $A'(x) > 0$. Also, you should have written $A > 1$ when $x > y$.
      $endgroup$
      – heropup
      Aug 26 '14 at 21:27




      $begingroup$
      I upvoted you because I think your solution is definitely the right way to go. However, it could use more detail: we want to be more rigorous about showing that $A'(x) > 0$. Also, you should have written $A > 1$ when $x > y$.
      $endgroup$
      – heropup
      Aug 26 '14 at 21:27












      $begingroup$
      @Krokop I think the downvoter is OP.. All of the answers in this thread have been downvoted.
      $endgroup$
      – Cameron Williams
      Aug 27 '14 at 3:09




      $begingroup$
      @Krokop I think the downvoter is OP.. All of the answers in this thread have been downvoted.
      $endgroup$
      – Cameron Williams
      Aug 27 '14 at 3:09












      $begingroup$
      @heropup Thanks for your comment, I edited to add some detail.
      $endgroup$
      – Krokop
      Aug 28 '14 at 12:07




      $begingroup$
      @heropup Thanks for your comment, I edited to add some detail.
      $endgroup$
      – Krokop
      Aug 28 '14 at 12:07












      $begingroup$
      @CameronWilliams It seems that he wants (always) a full detailed solution..
      $endgroup$
      – Krokop
      Aug 28 '14 at 12:12




      $begingroup$
      @CameronWilliams It seems that he wants (always) a full detailed solution..
      $endgroup$
      – Krokop
      Aug 28 '14 at 12:12











      3












      $begingroup$

      Multiplication:



      $y=x y-x sqrt{x^2+y}-y sqrt{-x+y^2}+sqrt{x^2+y} sqrt{-x+y^2} Rightarrow$



      $x sqrt{x^2+y}+y sqrt{-x+y^2}=sqrt{x^2+y} sqrt{-x+y^2}+xy-y$



      squaring both sides:



      $2 x y sqrt{x^2+y} sqrt{y^2-x}+x^4+x^2 y-x y^2+y^4=
      -x^3-x y+y^2-2 x y^2+2 x^2 y^2+y^3+(2 x y-2 y )sqrt{x^2+y} sqrt{-x+y^2} Rightarrow $



      simplifying:



      $x^4+x^3-2 x^2 y^2+x^2 y+x y^2+x y+y^4-y^3-y^2 = -2 y sqrt{x^2+y} sqrt{y^2-x}$



      Squaring again:



      $y^8 - 2 y^7 - 4 x^2 y^6 + 2 x y^6 - y^6 + 6 x^2 y^5 + 2 y^5 + 6 x^4 y^4 - 2 x^3 y^4 + 3 x^2 y^4 - 4 x y^4 + y^4 - 6 x^4 y^3 - 4 x^3 y^3 - 2 x y^3 - 4 x^6 y^2 - 2 x^5 y^2 + x^4 y^2 + x^2 y^2 + 2 x^6 y + 4 x^5 y + 2 x^4 y + x^8 + 2 x^7 + x^6=
      -4 x^3 y^2+4 x^2 y^4-4 x y^3+4 y^5$



      The only thing we have to check is how it factors as $(y+x) (p(x,y))$. Which gives us:



      $(x+y)^2(x^6-2 x^5 y+2 x^5-x^4 y^2-2 x^4 y+x^4+4 x^3 y^3+2 x^3 y-x^2 y^4-4 x^2 y^3-4 x^2 y^2+2 x^2 y-2 x y^5+6 x y^4+2 x y^3+y^6-2 y^5-y^4-2 y^3+y^2)=0$






      share|cite|improve this answer











      $endgroup$













      • $begingroup$
        Your last inequality is wrong. Try $x=0.8$ and $y=1$.
        $endgroup$
        – Michael Rozenberg
        Nov 29 '18 at 7:07
















      3












      $begingroup$

      Multiplication:



      $y=x y-x sqrt{x^2+y}-y sqrt{-x+y^2}+sqrt{x^2+y} sqrt{-x+y^2} Rightarrow$



      $x sqrt{x^2+y}+y sqrt{-x+y^2}=sqrt{x^2+y} sqrt{-x+y^2}+xy-y$



      squaring both sides:



      $2 x y sqrt{x^2+y} sqrt{y^2-x}+x^4+x^2 y-x y^2+y^4=
      -x^3-x y+y^2-2 x y^2+2 x^2 y^2+y^3+(2 x y-2 y )sqrt{x^2+y} sqrt{-x+y^2} Rightarrow $



      simplifying:



      $x^4+x^3-2 x^2 y^2+x^2 y+x y^2+x y+y^4-y^3-y^2 = -2 y sqrt{x^2+y} sqrt{y^2-x}$



      Squaring again:



      $y^8 - 2 y^7 - 4 x^2 y^6 + 2 x y^6 - y^6 + 6 x^2 y^5 + 2 y^5 + 6 x^4 y^4 - 2 x^3 y^4 + 3 x^2 y^4 - 4 x y^4 + y^4 - 6 x^4 y^3 - 4 x^3 y^3 - 2 x y^3 - 4 x^6 y^2 - 2 x^5 y^2 + x^4 y^2 + x^2 y^2 + 2 x^6 y + 4 x^5 y + 2 x^4 y + x^8 + 2 x^7 + x^6=
      -4 x^3 y^2+4 x^2 y^4-4 x y^3+4 y^5$



      The only thing we have to check is how it factors as $(y+x) (p(x,y))$. Which gives us:



      $(x+y)^2(x^6-2 x^5 y+2 x^5-x^4 y^2-2 x^4 y+x^4+4 x^3 y^3+2 x^3 y-x^2 y^4-4 x^2 y^3-4 x^2 y^2+2 x^2 y-2 x y^5+6 x y^4+2 x y^3+y^6-2 y^5-y^4-2 y^3+y^2)=0$






      share|cite|improve this answer











      $endgroup$













      • $begingroup$
        Your last inequality is wrong. Try $x=0.8$ and $y=1$.
        $endgroup$
        – Michael Rozenberg
        Nov 29 '18 at 7:07














      3












      3








      3





      $begingroup$

      Multiplication:



      $y=x y-x sqrt{x^2+y}-y sqrt{-x+y^2}+sqrt{x^2+y} sqrt{-x+y^2} Rightarrow$



      $x sqrt{x^2+y}+y sqrt{-x+y^2}=sqrt{x^2+y} sqrt{-x+y^2}+xy-y$



      squaring both sides:



      $2 x y sqrt{x^2+y} sqrt{y^2-x}+x^4+x^2 y-x y^2+y^4=
      -x^3-x y+y^2-2 x y^2+2 x^2 y^2+y^3+(2 x y-2 y )sqrt{x^2+y} sqrt{-x+y^2} Rightarrow $



      simplifying:



      $x^4+x^3-2 x^2 y^2+x^2 y+x y^2+x y+y^4-y^3-y^2 = -2 y sqrt{x^2+y} sqrt{y^2-x}$



      Squaring again:



      $y^8 - 2 y^7 - 4 x^2 y^6 + 2 x y^6 - y^6 + 6 x^2 y^5 + 2 y^5 + 6 x^4 y^4 - 2 x^3 y^4 + 3 x^2 y^4 - 4 x y^4 + y^4 - 6 x^4 y^3 - 4 x^3 y^3 - 2 x y^3 - 4 x^6 y^2 - 2 x^5 y^2 + x^4 y^2 + x^2 y^2 + 2 x^6 y + 4 x^5 y + 2 x^4 y + x^8 + 2 x^7 + x^6=
      -4 x^3 y^2+4 x^2 y^4-4 x y^3+4 y^5$



      The only thing we have to check is how it factors as $(y+x) (p(x,y))$. Which gives us:



      $(x+y)^2(x^6-2 x^5 y+2 x^5-x^4 y^2-2 x^4 y+x^4+4 x^3 y^3+2 x^3 y-x^2 y^4-4 x^2 y^3-4 x^2 y^2+2 x^2 y-2 x y^5+6 x y^4+2 x y^3+y^6-2 y^5-y^4-2 y^3+y^2)=0$






      share|cite|improve this answer











      $endgroup$



      Multiplication:



      $y=x y-x sqrt{x^2+y}-y sqrt{-x+y^2}+sqrt{x^2+y} sqrt{-x+y^2} Rightarrow$



      $x sqrt{x^2+y}+y sqrt{-x+y^2}=sqrt{x^2+y} sqrt{-x+y^2}+xy-y$



      squaring both sides:



      $2 x y sqrt{x^2+y} sqrt{y^2-x}+x^4+x^2 y-x y^2+y^4=
      -x^3-x y+y^2-2 x y^2+2 x^2 y^2+y^3+(2 x y-2 y )sqrt{x^2+y} sqrt{-x+y^2} Rightarrow $



      simplifying:



      $x^4+x^3-2 x^2 y^2+x^2 y+x y^2+x y+y^4-y^3-y^2 = -2 y sqrt{x^2+y} sqrt{y^2-x}$



      Squaring again:



      $y^8 - 2 y^7 - 4 x^2 y^6 + 2 x y^6 - y^6 + 6 x^2 y^5 + 2 y^5 + 6 x^4 y^4 - 2 x^3 y^4 + 3 x^2 y^4 - 4 x y^4 + y^4 - 6 x^4 y^3 - 4 x^3 y^3 - 2 x y^3 - 4 x^6 y^2 - 2 x^5 y^2 + x^4 y^2 + x^2 y^2 + 2 x^6 y + 4 x^5 y + 2 x^4 y + x^8 + 2 x^7 + x^6=
      -4 x^3 y^2+4 x^2 y^4-4 x y^3+4 y^5$



      The only thing we have to check is how it factors as $(y+x) (p(x,y))$. Which gives us:



      $(x+y)^2(x^6-2 x^5 y+2 x^5-x^4 y^2-2 x^4 y+x^4+4 x^3 y^3+2 x^3 y-x^2 y^4-4 x^2 y^3-4 x^2 y^2+2 x^2 y-2 x y^5+6 x y^4+2 x y^3+y^6-2 y^5-y^4-2 y^3+y^2)=0$







      share|cite|improve this answer














      share|cite|improve this answer



      share|cite|improve this answer








      edited Aug 27 '14 at 2:56

























      answered Aug 26 '14 at 21:10









      BuddhaBuddha

      566714




      566714












      • $begingroup$
        Your last inequality is wrong. Try $x=0.8$ and $y=1$.
        $endgroup$
        – Michael Rozenberg
        Nov 29 '18 at 7:07


















      • $begingroup$
        Your last inequality is wrong. Try $x=0.8$ and $y=1$.
        $endgroup$
        – Michael Rozenberg
        Nov 29 '18 at 7:07
















      $begingroup$
      Your last inequality is wrong. Try $x=0.8$ and $y=1$.
      $endgroup$
      – Michael Rozenberg
      Nov 29 '18 at 7:07




      $begingroup$
      Your last inequality is wrong. Try $x=0.8$ and $y=1$.
      $endgroup$
      – Michael Rozenberg
      Nov 29 '18 at 7:07











      2












      $begingroup$

      In the following proof we divide the $(x,y)$ plane into regions (see diagram) and show that each region can contain no solutions except on the line $x + y = 0$.



      enter image description here



      Let
      $$
      F(x,y) = U(x,y)V(x,y) – y
      $$
      where
      $$
      U(x,y) = sqrt{y^2 – x} ,, – x
      \V(x,y) = sqrt{x^2 + y} ,, – y
      $$



      Then solutions satisfy
      $$
      F(x,y) = 0
      $$



      Substituting $y=-x$ shows that $x+y=0$ is a solution for all x.



      U is nonreal where $x>y^2$ (regions A and C in the diagram, bounded by red lines) and V is nonreal in the region $y<-x^2$ (regions B and C, also bounded by red lines). In these regions F is nonreal except possibly in region C, where U and V are both unreal. But there the condition for F to be real reduces to $x+y=0$, a subset of the known solution.



      The following statements and deductions relate to the other regions of the $(x,y)$ plane, where $U$ and $V$ are real.



      $U<0 Leftrightarrow x>tfrac{1}{2}(-1 + sqrt{1 + 4y^2})$ (regions I, J).



      $V<0 Leftrightarrow y>tfrac{1}{2}(1 + sqrt{1 + 4x^2})$ (all regions except F, G).



      $U_{x} < 0$ (regions D-K)



      $V_{x} < 0 Leftrightarrow x < 0$ (regions D, E, F)



      $U_{xx} < 0$ (regions D-K)



      $V_{xx} < 0 Leftrightarrow y < 0$ (regions D, J, K)



      where a subscript x denotes partial differentiation with respect to x.



      On the diagram the lines on which $U=0$ and $V=0$ are coloured green and blue, respectively. It is easily shown that $F$ is nonzero on all the coloured lines (with sign as indicated) except at $(0,0)$ and $(1,-1)$. These lines delimit, but are excluded from, the regions A-K.



      From the results above we can make the following deductions.



      In region D:
      $$
      U>0, V>0, U_{x}<0, V_{x}<0
      \F_{x} = UV_{x} + VU_{x} < 0
      $$
      This region is bounded on the right by the line $y<-x^2$, on which $F>0$. So $F>0$ throughout region D and it can contain no solutions.



      In region E:
      $$
      U>0, V>0, U_{x}<0, V_{x}<0
      \F_{x} < 0
      $$
      so here there can be no solutions other than those known to exist on the line $y=-x$.



      In region F:
      $$
      U>0, V<0, U_{x}<0, V_{x}<0, U_{xx}<0, V_{xx}>0
      \F_{xx} = UV_{xx} + VU_{xx} + 2U_{x}V_{x} > 0
      $$
      This region is bounded on the left by the line $V=0$ and on the right by the line $x = 0$, and on both these lines $F<0$. So the positive second derivative $F_{xx}$ means there can be no solutions $F=0$ in this region.



      In a similar way, solutions can be ruled out for the following regions:



      In region G, bounded on right by the line $V=0$ on which $F<0$:
      $$
      U>0, V<0, U_{x}<0, V_{x}>0
      \F_{x}>0
      $$



      In region I, bounded on left by the line $U=0$ on which $F<0$:
      $$
      U<0, V>0, U_{x}<0, V_{x}>0
      \F_{x} < 0
      $$



      In region J, containing a segment of the known solution line $x+y=0$ on which $F=0$:
      $$
      U<0, V>0, U_{x}<0, V_{x}>0
      \F_{x} < 0
      $$



      In region K, bounded on the left and right by lines on which $F>0$:
      $$
      U>0, V>0, U_{x}<0, V_{x}>0, U_{xx}<0, V_{xx}<0
      \F_{xx} = UV_{xx} + VU_{xx} + 2U_{x}V_{x} < 0
      $$



      Finally, in region H:
      $$
      U>0, V>0, U_{x}<0, V_{x}>0
      $$
      and we note that $U_{x}<0$ in region G also, so for a given $y$,
      $U<U_{max}$, where $U_{max} = U(0,y) = y$



      For the same value of y, $V<V_{max}$, where $V_{max} = V(X,y)$, and X is the value of x on the right-hand boundary of the region. On this boundary, $y=sqrt{X^2+X}$, so
      $$
      V_{max} = V(X,y) = sqrt{X^2+y} , – y
      < sqrt{X^2+X+y} , - y = sqrt{y^2 + y} ,, – y < tfrac{1}{2}.
      $$



      Therefore
      $$
      F = UV – y < U_{max} V_{max} – y < y tfrac{1}{2} – y = , –tfrac{1}{2} y < 0
      $$
      which completes the proof that there are no solutions other than $x+y = 0$.






      share|cite|improve this answer











      $endgroup$













      • $begingroup$
        Are you sure that this is a solution?
        $endgroup$
        – Michael Rozenberg
        Nov 29 '18 at 7:33


















      2












      $begingroup$

      In the following proof we divide the $(x,y)$ plane into regions (see diagram) and show that each region can contain no solutions except on the line $x + y = 0$.



      enter image description here



      Let
      $$
      F(x,y) = U(x,y)V(x,y) – y
      $$
      where
      $$
      U(x,y) = sqrt{y^2 – x} ,, – x
      \V(x,y) = sqrt{x^2 + y} ,, – y
      $$



      Then solutions satisfy
      $$
      F(x,y) = 0
      $$



      Substituting $y=-x$ shows that $x+y=0$ is a solution for all x.



      U is nonreal where $x>y^2$ (regions A and C in the diagram, bounded by red lines) and V is nonreal in the region $y<-x^2$ (regions B and C, also bounded by red lines). In these regions F is nonreal except possibly in region C, where U and V are both unreal. But there the condition for F to be real reduces to $x+y=0$, a subset of the known solution.



      The following statements and deductions relate to the other regions of the $(x,y)$ plane, where $U$ and $V$ are real.



      $U<0 Leftrightarrow x>tfrac{1}{2}(-1 + sqrt{1 + 4y^2})$ (regions I, J).



      $V<0 Leftrightarrow y>tfrac{1}{2}(1 + sqrt{1 + 4x^2})$ (all regions except F, G).



      $U_{x} < 0$ (regions D-K)



      $V_{x} < 0 Leftrightarrow x < 0$ (regions D, E, F)



      $U_{xx} < 0$ (regions D-K)



      $V_{xx} < 0 Leftrightarrow y < 0$ (regions D, J, K)



      where a subscript x denotes partial differentiation with respect to x.



      On the diagram the lines on which $U=0$ and $V=0$ are coloured green and blue, respectively. It is easily shown that $F$ is nonzero on all the coloured lines (with sign as indicated) except at $(0,0)$ and $(1,-1)$. These lines delimit, but are excluded from, the regions A-K.



      From the results above we can make the following deductions.



      In region D:
      $$
      U>0, V>0, U_{x}<0, V_{x}<0
      \F_{x} = UV_{x} + VU_{x} < 0
      $$
      This region is bounded on the right by the line $y<-x^2$, on which $F>0$. So $F>0$ throughout region D and it can contain no solutions.



      In region E:
      $$
      U>0, V>0, U_{x}<0, V_{x}<0
      \F_{x} < 0
      $$
      so here there can be no solutions other than those known to exist on the line $y=-x$.



      In region F:
      $$
      U>0, V<0, U_{x}<0, V_{x}<0, U_{xx}<0, V_{xx}>0
      \F_{xx} = UV_{xx} + VU_{xx} + 2U_{x}V_{x} > 0
      $$
      This region is bounded on the left by the line $V=0$ and on the right by the line $x = 0$, and on both these lines $F<0$. So the positive second derivative $F_{xx}$ means there can be no solutions $F=0$ in this region.



      In a similar way, solutions can be ruled out for the following regions:



      In region G, bounded on right by the line $V=0$ on which $F<0$:
      $$
      U>0, V<0, U_{x}<0, V_{x}>0
      \F_{x}>0
      $$



      In region I, bounded on left by the line $U=0$ on which $F<0$:
      $$
      U<0, V>0, U_{x}<0, V_{x}>0
      \F_{x} < 0
      $$



      In region J, containing a segment of the known solution line $x+y=0$ on which $F=0$:
      $$
      U<0, V>0, U_{x}<0, V_{x}>0
      \F_{x} < 0
      $$



      In region K, bounded on the left and right by lines on which $F>0$:
      $$
      U>0, V>0, U_{x}<0, V_{x}>0, U_{xx}<0, V_{xx}<0
      \F_{xx} = UV_{xx} + VU_{xx} + 2U_{x}V_{x} < 0
      $$



      Finally, in region H:
      $$
      U>0, V>0, U_{x}<0, V_{x}>0
      $$
      and we note that $U_{x}<0$ in region G also, so for a given $y$,
      $U<U_{max}$, where $U_{max} = U(0,y) = y$



      For the same value of y, $V<V_{max}$, where $V_{max} = V(X,y)$, and X is the value of x on the right-hand boundary of the region. On this boundary, $y=sqrt{X^2+X}$, so
      $$
      V_{max} = V(X,y) = sqrt{X^2+y} , – y
      < sqrt{X^2+X+y} , - y = sqrt{y^2 + y} ,, – y < tfrac{1}{2}.
      $$



      Therefore
      $$
      F = UV – y < U_{max} V_{max} – y < y tfrac{1}{2} – y = , –tfrac{1}{2} y < 0
      $$
      which completes the proof that there are no solutions other than $x+y = 0$.






      share|cite|improve this answer











      $endgroup$













      • $begingroup$
        Are you sure that this is a solution?
        $endgroup$
        – Michael Rozenberg
        Nov 29 '18 at 7:33
















      2












      2








      2





      $begingroup$

      In the following proof we divide the $(x,y)$ plane into regions (see diagram) and show that each region can contain no solutions except on the line $x + y = 0$.



      enter image description here



      Let
      $$
      F(x,y) = U(x,y)V(x,y) – y
      $$
      where
      $$
      U(x,y) = sqrt{y^2 – x} ,, – x
      \V(x,y) = sqrt{x^2 + y} ,, – y
      $$



      Then solutions satisfy
      $$
      F(x,y) = 0
      $$



      Substituting $y=-x$ shows that $x+y=0$ is a solution for all x.



      U is nonreal where $x>y^2$ (regions A and C in the diagram, bounded by red lines) and V is nonreal in the region $y<-x^2$ (regions B and C, also bounded by red lines). In these regions F is nonreal except possibly in region C, where U and V are both unreal. But there the condition for F to be real reduces to $x+y=0$, a subset of the known solution.



      The following statements and deductions relate to the other regions of the $(x,y)$ plane, where $U$ and $V$ are real.



      $U<0 Leftrightarrow x>tfrac{1}{2}(-1 + sqrt{1 + 4y^2})$ (regions I, J).



      $V<0 Leftrightarrow y>tfrac{1}{2}(1 + sqrt{1 + 4x^2})$ (all regions except F, G).



      $U_{x} < 0$ (regions D-K)



      $V_{x} < 0 Leftrightarrow x < 0$ (regions D, E, F)



      $U_{xx} < 0$ (regions D-K)



      $V_{xx} < 0 Leftrightarrow y < 0$ (regions D, J, K)



      where a subscript x denotes partial differentiation with respect to x.



      On the diagram the lines on which $U=0$ and $V=0$ are coloured green and blue, respectively. It is easily shown that $F$ is nonzero on all the coloured lines (with sign as indicated) except at $(0,0)$ and $(1,-1)$. These lines delimit, but are excluded from, the regions A-K.



      From the results above we can make the following deductions.



      In region D:
      $$
      U>0, V>0, U_{x}<0, V_{x}<0
      \F_{x} = UV_{x} + VU_{x} < 0
      $$
      This region is bounded on the right by the line $y<-x^2$, on which $F>0$. So $F>0$ throughout region D and it can contain no solutions.



      In region E:
      $$
      U>0, V>0, U_{x}<0, V_{x}<0
      \F_{x} < 0
      $$
      so here there can be no solutions other than those known to exist on the line $y=-x$.



      In region F:
      $$
      U>0, V<0, U_{x}<0, V_{x}<0, U_{xx}<0, V_{xx}>0
      \F_{xx} = UV_{xx} + VU_{xx} + 2U_{x}V_{x} > 0
      $$
      This region is bounded on the left by the line $V=0$ and on the right by the line $x = 0$, and on both these lines $F<0$. So the positive second derivative $F_{xx}$ means there can be no solutions $F=0$ in this region.



      In a similar way, solutions can be ruled out for the following regions:



      In region G, bounded on right by the line $V=0$ on which $F<0$:
      $$
      U>0, V<0, U_{x}<0, V_{x}>0
      \F_{x}>0
      $$



      In region I, bounded on left by the line $U=0$ on which $F<0$:
      $$
      U<0, V>0, U_{x}<0, V_{x}>0
      \F_{x} < 0
      $$



      In region J, containing a segment of the known solution line $x+y=0$ on which $F=0$:
      $$
      U<0, V>0, U_{x}<0, V_{x}>0
      \F_{x} < 0
      $$



      In region K, bounded on the left and right by lines on which $F>0$:
      $$
      U>0, V>0, U_{x}<0, V_{x}>0, U_{xx}<0, V_{xx}<0
      \F_{xx} = UV_{xx} + VU_{xx} + 2U_{x}V_{x} < 0
      $$



      Finally, in region H:
      $$
      U>0, V>0, U_{x}<0, V_{x}>0
      $$
      and we note that $U_{x}<0$ in region G also, so for a given $y$,
      $U<U_{max}$, where $U_{max} = U(0,y) = y$



      For the same value of y, $V<V_{max}$, where $V_{max} = V(X,y)$, and X is the value of x on the right-hand boundary of the region. On this boundary, $y=sqrt{X^2+X}$, so
      $$
      V_{max} = V(X,y) = sqrt{X^2+y} , – y
      < sqrt{X^2+X+y} , - y = sqrt{y^2 + y} ,, – y < tfrac{1}{2}.
      $$



      Therefore
      $$
      F = UV – y < U_{max} V_{max} – y < y tfrac{1}{2} – y = , –tfrac{1}{2} y < 0
      $$
      which completes the proof that there are no solutions other than $x+y = 0$.






      share|cite|improve this answer











      $endgroup$



      In the following proof we divide the $(x,y)$ plane into regions (see diagram) and show that each region can contain no solutions except on the line $x + y = 0$.



      enter image description here



      Let
      $$
      F(x,y) = U(x,y)V(x,y) – y
      $$
      where
      $$
      U(x,y) = sqrt{y^2 – x} ,, – x
      \V(x,y) = sqrt{x^2 + y} ,, – y
      $$



      Then solutions satisfy
      $$
      F(x,y) = 0
      $$



      Substituting $y=-x$ shows that $x+y=0$ is a solution for all x.



      U is nonreal where $x>y^2$ (regions A and C in the diagram, bounded by red lines) and V is nonreal in the region $y<-x^2$ (regions B and C, also bounded by red lines). In these regions F is nonreal except possibly in region C, where U and V are both unreal. But there the condition for F to be real reduces to $x+y=0$, a subset of the known solution.



      The following statements and deductions relate to the other regions of the $(x,y)$ plane, where $U$ and $V$ are real.



      $U<0 Leftrightarrow x>tfrac{1}{2}(-1 + sqrt{1 + 4y^2})$ (regions I, J).



      $V<0 Leftrightarrow y>tfrac{1}{2}(1 + sqrt{1 + 4x^2})$ (all regions except F, G).



      $U_{x} < 0$ (regions D-K)



      $V_{x} < 0 Leftrightarrow x < 0$ (regions D, E, F)



      $U_{xx} < 0$ (regions D-K)



      $V_{xx} < 0 Leftrightarrow y < 0$ (regions D, J, K)



      where a subscript x denotes partial differentiation with respect to x.



      On the diagram the lines on which $U=0$ and $V=0$ are coloured green and blue, respectively. It is easily shown that $F$ is nonzero on all the coloured lines (with sign as indicated) except at $(0,0)$ and $(1,-1)$. These lines delimit, but are excluded from, the regions A-K.



      From the results above we can make the following deductions.



      In region D:
      $$
      U>0, V>0, U_{x}<0, V_{x}<0
      \F_{x} = UV_{x} + VU_{x} < 0
      $$
      This region is bounded on the right by the line $y<-x^2$, on which $F>0$. So $F>0$ throughout region D and it can contain no solutions.



      In region E:
      $$
      U>0, V>0, U_{x}<0, V_{x}<0
      \F_{x} < 0
      $$
      so here there can be no solutions other than those known to exist on the line $y=-x$.



      In region F:
      $$
      U>0, V<0, U_{x}<0, V_{x}<0, U_{xx}<0, V_{xx}>0
      \F_{xx} = UV_{xx} + VU_{xx} + 2U_{x}V_{x} > 0
      $$
      This region is bounded on the left by the line $V=0$ and on the right by the line $x = 0$, and on both these lines $F<0$. So the positive second derivative $F_{xx}$ means there can be no solutions $F=0$ in this region.



      In a similar way, solutions can be ruled out for the following regions:



      In region G, bounded on right by the line $V=0$ on which $F<0$:
      $$
      U>0, V<0, U_{x}<0, V_{x}>0
      \F_{x}>0
      $$



      In region I, bounded on left by the line $U=0$ on which $F<0$:
      $$
      U<0, V>0, U_{x}<0, V_{x}>0
      \F_{x} < 0
      $$



      In region J, containing a segment of the known solution line $x+y=0$ on which $F=0$:
      $$
      U<0, V>0, U_{x}<0, V_{x}>0
      \F_{x} < 0
      $$



      In region K, bounded on the left and right by lines on which $F>0$:
      $$
      U>0, V>0, U_{x}<0, V_{x}>0, U_{xx}<0, V_{xx}<0
      \F_{xx} = UV_{xx} + VU_{xx} + 2U_{x}V_{x} < 0
      $$



      Finally, in region H:
      $$
      U>0, V>0, U_{x}<0, V_{x}>0
      $$
      and we note that $U_{x}<0$ in region G also, so for a given $y$,
      $U<U_{max}$, where $U_{max} = U(0,y) = y$



      For the same value of y, $V<V_{max}$, where $V_{max} = V(X,y)$, and X is the value of x on the right-hand boundary of the region. On this boundary, $y=sqrt{X^2+X}$, so
      $$
      V_{max} = V(X,y) = sqrt{X^2+y} , – y
      < sqrt{X^2+X+y} , - y = sqrt{y^2 + y} ,, – y < tfrac{1}{2}.
      $$



      Therefore
      $$
      F = UV – y < U_{max} V_{max} – y < y tfrac{1}{2} – y = , –tfrac{1}{2} y < 0
      $$
      which completes the proof that there are no solutions other than $x+y = 0$.







      share|cite|improve this answer














      share|cite|improve this answer



      share|cite|improve this answer








      edited Aug 31 '14 at 10:06

























      answered Aug 22 '14 at 22:06









      MartinGMartinG

      853511




      853511












      • $begingroup$
        Are you sure that this is a solution?
        $endgroup$
        – Michael Rozenberg
        Nov 29 '18 at 7:33




















      • $begingroup$
        Are you sure that this is a solution?
        $endgroup$
        – Michael Rozenberg
        Nov 29 '18 at 7:33


















      $begingroup$
      Are you sure that this is a solution?
      $endgroup$
      – Michael Rozenberg
      Nov 29 '18 at 7:33






      $begingroup$
      Are you sure that this is a solution?
      $endgroup$
      – Michael Rozenberg
      Nov 29 '18 at 7:33













      2












      $begingroup$

      Assuming continuity in the area interval $ (0 < x < 1 )$ and $ (0 > y > -1 ) $ would create problems as $x$ and $y$ are not always real in these areas.



      Nay, union of inside parabola areas of $ y_1 = - x^2 $ and $ y_2 = sqrt{x} $ would violate $ x + y = 0, $ which is only the common chord of intersection of $ y_1,y_2$. So the shown line joining $(0,0)$ to $(1,-1)$ does not exist as real.
      E.g., $(frac12, -frac12)$ does not lie on the
      common line.



      enter image description here






      share|cite|improve this answer











      $endgroup$


















        2












        $begingroup$

        Assuming continuity in the area interval $ (0 < x < 1 )$ and $ (0 > y > -1 ) $ would create problems as $x$ and $y$ are not always real in these areas.



        Nay, union of inside parabola areas of $ y_1 = - x^2 $ and $ y_2 = sqrt{x} $ would violate $ x + y = 0, $ which is only the common chord of intersection of $ y_1,y_2$. So the shown line joining $(0,0)$ to $(1,-1)$ does not exist as real.
        E.g., $(frac12, -frac12)$ does not lie on the
        common line.



        enter image description here






        share|cite|improve this answer











        $endgroup$
















          2












          2








          2





          $begingroup$

          Assuming continuity in the area interval $ (0 < x < 1 )$ and $ (0 > y > -1 ) $ would create problems as $x$ and $y$ are not always real in these areas.



          Nay, union of inside parabola areas of $ y_1 = - x^2 $ and $ y_2 = sqrt{x} $ would violate $ x + y = 0, $ which is only the common chord of intersection of $ y_1,y_2$. So the shown line joining $(0,0)$ to $(1,-1)$ does not exist as real.
          E.g., $(frac12, -frac12)$ does not lie on the
          common line.



          enter image description here






          share|cite|improve this answer











          $endgroup$



          Assuming continuity in the area interval $ (0 < x < 1 )$ and $ (0 > y > -1 ) $ would create problems as $x$ and $y$ are not always real in these areas.



          Nay, union of inside parabola areas of $ y_1 = - x^2 $ and $ y_2 = sqrt{x} $ would violate $ x + y = 0, $ which is only the common chord of intersection of $ y_1,y_2$. So the shown line joining $(0,0)$ to $(1,-1)$ does not exist as real.
          E.g., $(frac12, -frac12)$ does not lie on the
          common line.



          enter image description here







          share|cite|improve this answer














          share|cite|improve this answer



          share|cite|improve this answer








          edited Sep 7 '17 at 15:41

























          answered Aug 26 '14 at 15:28









          NarasimhamNarasimham

          20.7k52158




          20.7k52158























              1












              $begingroup$

              This is not a solution, but brute force can be used to remove the radicals. Let $A=y^2-x$ and $B=x^2+y$. We have



              $$sqrt{AB}-ysqrt{A}-xsqrt{B}+xy=y$$



              Isolating $sqrt{AB}$ and squaring both sides:



              $$sqrt{AB}=ysqrt{A}+xsqrt{B}+y(1-x)quad(1)$$
              $$AB=y^2A+x^2B+y^2(1-x)^2+2xysqrt{AB}+2y^2(1-x)sqrt{A}+2xy(1-x)sqrt{B}$$



              (1) allows us to remove $sqrt{AB}$. We do this and also recall what $A$ and $B$ equal.



              $$(y^2-x)(x^2+y)=y^2(y^2-x)+x^2(x^2+y)+y^2(1-x)^2+2xyleft(ysqrt{A}+xsqrt{B}+y(1-x)right)+2y^2(1-x)sqrt{A}+2xy(1-x)sqrt{B}$$



              Group $sqrt{A}$ and $sqrt{B}$ terms, then rearrange a bit:



              $$(y^2-x)(x^2+y)=y^2(y^2-x)+x^2(x^2+y)+y^2(1-x)^2+2xy^2(1-x)+2y^2sqrt{A}+2xysqrt{B}$$
              $$x^2y^2+y^3-x^3-xy=y^4-xy^2+x^4+x^2y+y^2-2xy^2+x^2y^2+2xy^2-2x^2y^2+2yleft(ysqrt{A}+xsqrt{B}right)$$
              $$y^3-x^3-xy=y^4-xy^2+x^4+x^2y+y^2-2x^2y^2+2yleft(ysqrt{A}+xsqrt{B}right)$$



              (1) allows us to sub out the quantity in parentheses:



              $$y^3-x^3-xy=y^4-xy^2+x^4+x^2y+y^2-2x^2y^2+2yleft(y(x-1)+sqrt{AB}right)$$
              $$y^3-x^3-xy=y^4-xy^2+x^4+x^2y+y^2-2x^2y^2+2y^2(x-1)+2ysqrt{AB}$$
              $$y^3-x^3-xy=y^4+xy^2+x^4+x^2y-y^2-2x^2y^2+2ysqrt{AB}$$
              $$y^3-x^3-xy-y^4-xy^2-x^4-x^2y+y^2+2x^2y^2=2ysqrt{AB}$$



              Squaring both sides, we've reached a goal of no longer having radicals.



              $$(y^3-x^3-xy-y^4-xy^2-x^4-x^2y+y^2+2x^2y^2)^2=4y^2(y^2-x)(x^2+y)$$



              I had a CAS expand this, move it all to one side, and then, as expected, $(x+y)$ factors out of it (twice).



              $$(x+y)^2 p(x,y)=0$$



              where $$p(x,y)=x^6-2 x^5 y+2 x^5-x^4 y^2-2 x^4 y+x^4+4 x^3 y^3+2 x^3 y-x^2 y^4-4 x^2 y^3-4 x^2 y^2+2 x^2 y-2 x y^5+6 x y^4+2 x y^3+y^6-2 y^5-y^4-2 y^3+y^2$$



              is a monster. It would be sufficient to show that $p(x,y)$ is never $0$ in the region of the plane where both $sqrt{A}$ and $sqrt{B}$ are defined aside from points along $x+y=0$ (like $(0,0)$). This is a pretty messy polynomial, but at least it's a polynomial.





              EDIT: This approach seems to be useless; a CAS plot of the zero set of $p$ has several components, all of which are in the region where $sqrt{A}$ and $sqrt{B}$ are defined. They must be extraneous solutions from the squaring that was done twice.






              share|cite|improve this answer











              $endgroup$


















                1












                $begingroup$

                This is not a solution, but brute force can be used to remove the radicals. Let $A=y^2-x$ and $B=x^2+y$. We have



                $$sqrt{AB}-ysqrt{A}-xsqrt{B}+xy=y$$



                Isolating $sqrt{AB}$ and squaring both sides:



                $$sqrt{AB}=ysqrt{A}+xsqrt{B}+y(1-x)quad(1)$$
                $$AB=y^2A+x^2B+y^2(1-x)^2+2xysqrt{AB}+2y^2(1-x)sqrt{A}+2xy(1-x)sqrt{B}$$



                (1) allows us to remove $sqrt{AB}$. We do this and also recall what $A$ and $B$ equal.



                $$(y^2-x)(x^2+y)=y^2(y^2-x)+x^2(x^2+y)+y^2(1-x)^2+2xyleft(ysqrt{A}+xsqrt{B}+y(1-x)right)+2y^2(1-x)sqrt{A}+2xy(1-x)sqrt{B}$$



                Group $sqrt{A}$ and $sqrt{B}$ terms, then rearrange a bit:



                $$(y^2-x)(x^2+y)=y^2(y^2-x)+x^2(x^2+y)+y^2(1-x)^2+2xy^2(1-x)+2y^2sqrt{A}+2xysqrt{B}$$
                $$x^2y^2+y^3-x^3-xy=y^4-xy^2+x^4+x^2y+y^2-2xy^2+x^2y^2+2xy^2-2x^2y^2+2yleft(ysqrt{A}+xsqrt{B}right)$$
                $$y^3-x^3-xy=y^4-xy^2+x^4+x^2y+y^2-2x^2y^2+2yleft(ysqrt{A}+xsqrt{B}right)$$



                (1) allows us to sub out the quantity in parentheses:



                $$y^3-x^3-xy=y^4-xy^2+x^4+x^2y+y^2-2x^2y^2+2yleft(y(x-1)+sqrt{AB}right)$$
                $$y^3-x^3-xy=y^4-xy^2+x^4+x^2y+y^2-2x^2y^2+2y^2(x-1)+2ysqrt{AB}$$
                $$y^3-x^3-xy=y^4+xy^2+x^4+x^2y-y^2-2x^2y^2+2ysqrt{AB}$$
                $$y^3-x^3-xy-y^4-xy^2-x^4-x^2y+y^2+2x^2y^2=2ysqrt{AB}$$



                Squaring both sides, we've reached a goal of no longer having radicals.



                $$(y^3-x^3-xy-y^4-xy^2-x^4-x^2y+y^2+2x^2y^2)^2=4y^2(y^2-x)(x^2+y)$$



                I had a CAS expand this, move it all to one side, and then, as expected, $(x+y)$ factors out of it (twice).



                $$(x+y)^2 p(x,y)=0$$



                where $$p(x,y)=x^6-2 x^5 y+2 x^5-x^4 y^2-2 x^4 y+x^4+4 x^3 y^3+2 x^3 y-x^2 y^4-4 x^2 y^3-4 x^2 y^2+2 x^2 y-2 x y^5+6 x y^4+2 x y^3+y^6-2 y^5-y^4-2 y^3+y^2$$



                is a monster. It would be sufficient to show that $p(x,y)$ is never $0$ in the region of the plane where both $sqrt{A}$ and $sqrt{B}$ are defined aside from points along $x+y=0$ (like $(0,0)$). This is a pretty messy polynomial, but at least it's a polynomial.





                EDIT: This approach seems to be useless; a CAS plot of the zero set of $p$ has several components, all of which are in the region where $sqrt{A}$ and $sqrt{B}$ are defined. They must be extraneous solutions from the squaring that was done twice.






                share|cite|improve this answer











                $endgroup$
















                  1












                  1








                  1





                  $begingroup$

                  This is not a solution, but brute force can be used to remove the radicals. Let $A=y^2-x$ and $B=x^2+y$. We have



                  $$sqrt{AB}-ysqrt{A}-xsqrt{B}+xy=y$$



                  Isolating $sqrt{AB}$ and squaring both sides:



                  $$sqrt{AB}=ysqrt{A}+xsqrt{B}+y(1-x)quad(1)$$
                  $$AB=y^2A+x^2B+y^2(1-x)^2+2xysqrt{AB}+2y^2(1-x)sqrt{A}+2xy(1-x)sqrt{B}$$



                  (1) allows us to remove $sqrt{AB}$. We do this and also recall what $A$ and $B$ equal.



                  $$(y^2-x)(x^2+y)=y^2(y^2-x)+x^2(x^2+y)+y^2(1-x)^2+2xyleft(ysqrt{A}+xsqrt{B}+y(1-x)right)+2y^2(1-x)sqrt{A}+2xy(1-x)sqrt{B}$$



                  Group $sqrt{A}$ and $sqrt{B}$ terms, then rearrange a bit:



                  $$(y^2-x)(x^2+y)=y^2(y^2-x)+x^2(x^2+y)+y^2(1-x)^2+2xy^2(1-x)+2y^2sqrt{A}+2xysqrt{B}$$
                  $$x^2y^2+y^3-x^3-xy=y^4-xy^2+x^4+x^2y+y^2-2xy^2+x^2y^2+2xy^2-2x^2y^2+2yleft(ysqrt{A}+xsqrt{B}right)$$
                  $$y^3-x^3-xy=y^4-xy^2+x^4+x^2y+y^2-2x^2y^2+2yleft(ysqrt{A}+xsqrt{B}right)$$



                  (1) allows us to sub out the quantity in parentheses:



                  $$y^3-x^3-xy=y^4-xy^2+x^4+x^2y+y^2-2x^2y^2+2yleft(y(x-1)+sqrt{AB}right)$$
                  $$y^3-x^3-xy=y^4-xy^2+x^4+x^2y+y^2-2x^2y^2+2y^2(x-1)+2ysqrt{AB}$$
                  $$y^3-x^3-xy=y^4+xy^2+x^4+x^2y-y^2-2x^2y^2+2ysqrt{AB}$$
                  $$y^3-x^3-xy-y^4-xy^2-x^4-x^2y+y^2+2x^2y^2=2ysqrt{AB}$$



                  Squaring both sides, we've reached a goal of no longer having radicals.



                  $$(y^3-x^3-xy-y^4-xy^2-x^4-x^2y+y^2+2x^2y^2)^2=4y^2(y^2-x)(x^2+y)$$



                  I had a CAS expand this, move it all to one side, and then, as expected, $(x+y)$ factors out of it (twice).



                  $$(x+y)^2 p(x,y)=0$$



                  where $$p(x,y)=x^6-2 x^5 y+2 x^5-x^4 y^2-2 x^4 y+x^4+4 x^3 y^3+2 x^3 y-x^2 y^4-4 x^2 y^3-4 x^2 y^2+2 x^2 y-2 x y^5+6 x y^4+2 x y^3+y^6-2 y^5-y^4-2 y^3+y^2$$



                  is a monster. It would be sufficient to show that $p(x,y)$ is never $0$ in the region of the plane where both $sqrt{A}$ and $sqrt{B}$ are defined aside from points along $x+y=0$ (like $(0,0)$). This is a pretty messy polynomial, but at least it's a polynomial.





                  EDIT: This approach seems to be useless; a CAS plot of the zero set of $p$ has several components, all of which are in the region where $sqrt{A}$ and $sqrt{B}$ are defined. They must be extraneous solutions from the squaring that was done twice.






                  share|cite|improve this answer











                  $endgroup$



                  This is not a solution, but brute force can be used to remove the radicals. Let $A=y^2-x$ and $B=x^2+y$. We have



                  $$sqrt{AB}-ysqrt{A}-xsqrt{B}+xy=y$$



                  Isolating $sqrt{AB}$ and squaring both sides:



                  $$sqrt{AB}=ysqrt{A}+xsqrt{B}+y(1-x)quad(1)$$
                  $$AB=y^2A+x^2B+y^2(1-x)^2+2xysqrt{AB}+2y^2(1-x)sqrt{A}+2xy(1-x)sqrt{B}$$



                  (1) allows us to remove $sqrt{AB}$. We do this and also recall what $A$ and $B$ equal.



                  $$(y^2-x)(x^2+y)=y^2(y^2-x)+x^2(x^2+y)+y^2(1-x)^2+2xyleft(ysqrt{A}+xsqrt{B}+y(1-x)right)+2y^2(1-x)sqrt{A}+2xy(1-x)sqrt{B}$$



                  Group $sqrt{A}$ and $sqrt{B}$ terms, then rearrange a bit:



                  $$(y^2-x)(x^2+y)=y^2(y^2-x)+x^2(x^2+y)+y^2(1-x)^2+2xy^2(1-x)+2y^2sqrt{A}+2xysqrt{B}$$
                  $$x^2y^2+y^3-x^3-xy=y^4-xy^2+x^4+x^2y+y^2-2xy^2+x^2y^2+2xy^2-2x^2y^2+2yleft(ysqrt{A}+xsqrt{B}right)$$
                  $$y^3-x^3-xy=y^4-xy^2+x^4+x^2y+y^2-2x^2y^2+2yleft(ysqrt{A}+xsqrt{B}right)$$



                  (1) allows us to sub out the quantity in parentheses:



                  $$y^3-x^3-xy=y^4-xy^2+x^4+x^2y+y^2-2x^2y^2+2yleft(y(x-1)+sqrt{AB}right)$$
                  $$y^3-x^3-xy=y^4-xy^2+x^4+x^2y+y^2-2x^2y^2+2y^2(x-1)+2ysqrt{AB}$$
                  $$y^3-x^3-xy=y^4+xy^2+x^4+x^2y-y^2-2x^2y^2+2ysqrt{AB}$$
                  $$y^3-x^3-xy-y^4-xy^2-x^4-x^2y+y^2+2x^2y^2=2ysqrt{AB}$$



                  Squaring both sides, we've reached a goal of no longer having radicals.



                  $$(y^3-x^3-xy-y^4-xy^2-x^4-x^2y+y^2+2x^2y^2)^2=4y^2(y^2-x)(x^2+y)$$



                  I had a CAS expand this, move it all to one side, and then, as expected, $(x+y)$ factors out of it (twice).



                  $$(x+y)^2 p(x,y)=0$$



                  where $$p(x,y)=x^6-2 x^5 y+2 x^5-x^4 y^2-2 x^4 y+x^4+4 x^3 y^3+2 x^3 y-x^2 y^4-4 x^2 y^3-4 x^2 y^2+2 x^2 y-2 x y^5+6 x y^4+2 x y^3+y^6-2 y^5-y^4-2 y^3+y^2$$



                  is a monster. It would be sufficient to show that $p(x,y)$ is never $0$ in the region of the plane where both $sqrt{A}$ and $sqrt{B}$ are defined aside from points along $x+y=0$ (like $(0,0)$). This is a pretty messy polynomial, but at least it's a polynomial.





                  EDIT: This approach seems to be useless; a CAS plot of the zero set of $p$ has several components, all of which are in the region where $sqrt{A}$ and $sqrt{B}$ are defined. They must be extraneous solutions from the squaring that was done twice.







                  share|cite|improve this answer














                  share|cite|improve this answer



                  share|cite|improve this answer








                  edited Oct 29 '13 at 15:08

























                  answered Oct 29 '13 at 5:36









                  alex.jordanalex.jordan

                  39k560120




                  39k560120























                      0












                      $begingroup$

                      A more generalised approach over my earlier post. This is not intended to be an exhaustive proof but an experimental one. Constructive comments are most welcome.



                      Let



                      $$sqrt{y^2-x}-x=Ay^nqquad cdots (1)\
                      sqrt{x^2+y}-y=frac {y^{1-n}}A qquad cdots (2)\$$
                      such that the original equation $$left(sqrt{y^2-x}-xright)left(sqrt{x^2+y}-yright)=y$$
                      is satisfied as required.



                      From $(1)$,



                      $$begin{align}
                      sqrt{y^2-x}&=x+Ay^n\
                      y^2-x&=x^2++2Axy^n+A^2y^{2n}\
                      y^{2n}A^2+2xy^nA+(x^2-y^2+x)&=0\
                      A^2+frac {2x}{y^n}A+frac{(x^2-y^2+x)}{y^{2n}}&=0qquad qquad qquad qquad cdots (3)
                      end{align}$$



                      From $(2)$,



                      $$begin{align}
                      sqrt{x^2+y}&=y+frac {y^{1-n}}A\
                      Asqrt{x^2+y}&=Ay+y^{1-n}\
                      A^2(x^2+y)&=A^2y^2+2Ay^{2-n}+y^{2(1-n)}\
                      (x^2-y^2+y)A^2-2y^{2-n}A-y^{2(1-n)}&=0\
                      A^2-frac{2y^{2-n}}{x^2-y^2+y}A-frac{y^{2(1-n)}}{x^2-y^2+y}&=0qquad qquad qquad cdots (4)
                      end{align}$$



                      Equating coefficients of $A^1$:
                      $$begin{align}frac{2x}{y^n}&=-frac{2y^{2-n}}{x^2-y^2+y}\
                      y^2&=-x(x^2-y^2+y)qquad qquad qquad qquad qquad qquad cdots (5)end{align}$$



                      Equating coefficients of $A^0$:



                      $$begin{align}frac{x^2-y^2+x}{y^{2n}}&=-frac{y^{2(1-n)}}{x^2-y^2+y}\
                      y^2&=-(x^2-y^2+x)(x^2-y^2+y)qquad cdots (6)end{align}$$



                      (5)=(6):
                      $$begin{align}x(x^2-y^2+y)&=(x^2-y^2+x)(x^2-y^2+y)\
                      (x^2-y^2)(x^2+y^2-y)&=0\
                      (x-y)(x+y)(x^2-y^2+y)&=0\
                      Rightarrow x-y=&0, x+y=0, x^2-y^2+y=0end{align}$$



                      Checking by substitution into the original equation shows that only
                      $$x+y=0$$
                      is valid.



                      This graph created on desmos.com might help illustrate the approach:



                      https://www.desmos.com/calculator/qrlbgbalix






                      share|cite|improve this answer











                      $endgroup$













                      • $begingroup$
                        It's an interesting approach, but I don't buy it. Equations (3) and (4) show us that $A$ is a solution to the two indicated quadratic equations. It does not follow that they must be the same equation!
                        $endgroup$
                        – Bungo
                        Aug 23 '14 at 6:12










                      • $begingroup$
                        @Bungo Thanks for your comments. $A$ is actually a parameter in this case, and defines a series of curves for each equation. However at the same value of $A$ both equations intersect at a point which satisfies the original equation. The locus of this point is the solution required. Setting both quadratic equations to be the same is essentially assigning the same value of $A$ for both equations.
                        $endgroup$
                        – hypergeometric
                        Aug 23 '14 at 9:10










                      • $begingroup$
                        I agree that this is valid until you square the equations. But after squaring, each equation has a second root in addition to $A$. How do we know that the second root in (3) is the same as the second root in (4)? Sorry if I'm being dense. To take a simpler example: if $x = 1$ then for any $c$ I can express this as $x-c = 1-c$. Squaring both sides, I get $x^2 - 2xc + c^2 = 1 - 2c + c^2$, or $x^2 - 2xc - (1 - 2c) = 0$. If I carry out this procedure with two different values of $c$, I get two valid equations with $1$ as a root, but the coefficients are not the same.
                        $endgroup$
                        – Bungo
                        Aug 23 '14 at 16:40










                      • $begingroup$
                        That's true. By equating coefficients we arrive at one particular solution, but there may be others, e.g. where the two quadratics are not the same but have one common root...would it be correct to say that? Any suggestions on other possible approaches?
                        $endgroup$
                        – hypergeometric
                        Aug 23 '14 at 16:58










                      • $begingroup$
                        Yes, that's what I was getting at. If two quadratic equations have BOTH roots in common then their coefficients are the same, up to a common factor. (Actually that's one more issue with your approach: $x^2 + 2x + 1 = 0$ and $2x^2 + 4x + 2 = 0$ have the same roots but the coefficients are not equal.) Sorry, I don't have any suggestions at the moment - the other answer is pretty thorny and I haven't read through it yet. :-)
                        $endgroup$
                        – Bungo
                        Aug 23 '14 at 17:03
















                      0












                      $begingroup$

                      A more generalised approach over my earlier post. This is not intended to be an exhaustive proof but an experimental one. Constructive comments are most welcome.



                      Let



                      $$sqrt{y^2-x}-x=Ay^nqquad cdots (1)\
                      sqrt{x^2+y}-y=frac {y^{1-n}}A qquad cdots (2)\$$
                      such that the original equation $$left(sqrt{y^2-x}-xright)left(sqrt{x^2+y}-yright)=y$$
                      is satisfied as required.



                      From $(1)$,



                      $$begin{align}
                      sqrt{y^2-x}&=x+Ay^n\
                      y^2-x&=x^2++2Axy^n+A^2y^{2n}\
                      y^{2n}A^2+2xy^nA+(x^2-y^2+x)&=0\
                      A^2+frac {2x}{y^n}A+frac{(x^2-y^2+x)}{y^{2n}}&=0qquad qquad qquad qquad cdots (3)
                      end{align}$$



                      From $(2)$,



                      $$begin{align}
                      sqrt{x^2+y}&=y+frac {y^{1-n}}A\
                      Asqrt{x^2+y}&=Ay+y^{1-n}\
                      A^2(x^2+y)&=A^2y^2+2Ay^{2-n}+y^{2(1-n)}\
                      (x^2-y^2+y)A^2-2y^{2-n}A-y^{2(1-n)}&=0\
                      A^2-frac{2y^{2-n}}{x^2-y^2+y}A-frac{y^{2(1-n)}}{x^2-y^2+y}&=0qquad qquad qquad cdots (4)
                      end{align}$$



                      Equating coefficients of $A^1$:
                      $$begin{align}frac{2x}{y^n}&=-frac{2y^{2-n}}{x^2-y^2+y}\
                      y^2&=-x(x^2-y^2+y)qquad qquad qquad qquad qquad qquad cdots (5)end{align}$$



                      Equating coefficients of $A^0$:



                      $$begin{align}frac{x^2-y^2+x}{y^{2n}}&=-frac{y^{2(1-n)}}{x^2-y^2+y}\
                      y^2&=-(x^2-y^2+x)(x^2-y^2+y)qquad cdots (6)end{align}$$



                      (5)=(6):
                      $$begin{align}x(x^2-y^2+y)&=(x^2-y^2+x)(x^2-y^2+y)\
                      (x^2-y^2)(x^2+y^2-y)&=0\
                      (x-y)(x+y)(x^2-y^2+y)&=0\
                      Rightarrow x-y=&0, x+y=0, x^2-y^2+y=0end{align}$$



                      Checking by substitution into the original equation shows that only
                      $$x+y=0$$
                      is valid.



                      This graph created on desmos.com might help illustrate the approach:



                      https://www.desmos.com/calculator/qrlbgbalix






                      share|cite|improve this answer











                      $endgroup$













                      • $begingroup$
                        It's an interesting approach, but I don't buy it. Equations (3) and (4) show us that $A$ is a solution to the two indicated quadratic equations. It does not follow that they must be the same equation!
                        $endgroup$
                        – Bungo
                        Aug 23 '14 at 6:12










                      • $begingroup$
                        @Bungo Thanks for your comments. $A$ is actually a parameter in this case, and defines a series of curves for each equation. However at the same value of $A$ both equations intersect at a point which satisfies the original equation. The locus of this point is the solution required. Setting both quadratic equations to be the same is essentially assigning the same value of $A$ for both equations.
                        $endgroup$
                        – hypergeometric
                        Aug 23 '14 at 9:10










                      • $begingroup$
                        I agree that this is valid until you square the equations. But after squaring, each equation has a second root in addition to $A$. How do we know that the second root in (3) is the same as the second root in (4)? Sorry if I'm being dense. To take a simpler example: if $x = 1$ then for any $c$ I can express this as $x-c = 1-c$. Squaring both sides, I get $x^2 - 2xc + c^2 = 1 - 2c + c^2$, or $x^2 - 2xc - (1 - 2c) = 0$. If I carry out this procedure with two different values of $c$, I get two valid equations with $1$ as a root, but the coefficients are not the same.
                        $endgroup$
                        – Bungo
                        Aug 23 '14 at 16:40










                      • $begingroup$
                        That's true. By equating coefficients we arrive at one particular solution, but there may be others, e.g. where the two quadratics are not the same but have one common root...would it be correct to say that? Any suggestions on other possible approaches?
                        $endgroup$
                        – hypergeometric
                        Aug 23 '14 at 16:58










                      • $begingroup$
                        Yes, that's what I was getting at. If two quadratic equations have BOTH roots in common then their coefficients are the same, up to a common factor. (Actually that's one more issue with your approach: $x^2 + 2x + 1 = 0$ and $2x^2 + 4x + 2 = 0$ have the same roots but the coefficients are not equal.) Sorry, I don't have any suggestions at the moment - the other answer is pretty thorny and I haven't read through it yet. :-)
                        $endgroup$
                        – Bungo
                        Aug 23 '14 at 17:03














                      0












                      0








                      0





                      $begingroup$

                      A more generalised approach over my earlier post. This is not intended to be an exhaustive proof but an experimental one. Constructive comments are most welcome.



                      Let



                      $$sqrt{y^2-x}-x=Ay^nqquad cdots (1)\
                      sqrt{x^2+y}-y=frac {y^{1-n}}A qquad cdots (2)\$$
                      such that the original equation $$left(sqrt{y^2-x}-xright)left(sqrt{x^2+y}-yright)=y$$
                      is satisfied as required.



                      From $(1)$,



                      $$begin{align}
                      sqrt{y^2-x}&=x+Ay^n\
                      y^2-x&=x^2++2Axy^n+A^2y^{2n}\
                      y^{2n}A^2+2xy^nA+(x^2-y^2+x)&=0\
                      A^2+frac {2x}{y^n}A+frac{(x^2-y^2+x)}{y^{2n}}&=0qquad qquad qquad qquad cdots (3)
                      end{align}$$



                      From $(2)$,



                      $$begin{align}
                      sqrt{x^2+y}&=y+frac {y^{1-n}}A\
                      Asqrt{x^2+y}&=Ay+y^{1-n}\
                      A^2(x^2+y)&=A^2y^2+2Ay^{2-n}+y^{2(1-n)}\
                      (x^2-y^2+y)A^2-2y^{2-n}A-y^{2(1-n)}&=0\
                      A^2-frac{2y^{2-n}}{x^2-y^2+y}A-frac{y^{2(1-n)}}{x^2-y^2+y}&=0qquad qquad qquad cdots (4)
                      end{align}$$



                      Equating coefficients of $A^1$:
                      $$begin{align}frac{2x}{y^n}&=-frac{2y^{2-n}}{x^2-y^2+y}\
                      y^2&=-x(x^2-y^2+y)qquad qquad qquad qquad qquad qquad cdots (5)end{align}$$



                      Equating coefficients of $A^0$:



                      $$begin{align}frac{x^2-y^2+x}{y^{2n}}&=-frac{y^{2(1-n)}}{x^2-y^2+y}\
                      y^2&=-(x^2-y^2+x)(x^2-y^2+y)qquad cdots (6)end{align}$$



                      (5)=(6):
                      $$begin{align}x(x^2-y^2+y)&=(x^2-y^2+x)(x^2-y^2+y)\
                      (x^2-y^2)(x^2+y^2-y)&=0\
                      (x-y)(x+y)(x^2-y^2+y)&=0\
                      Rightarrow x-y=&0, x+y=0, x^2-y^2+y=0end{align}$$



                      Checking by substitution into the original equation shows that only
                      $$x+y=0$$
                      is valid.



                      This graph created on desmos.com might help illustrate the approach:



                      https://www.desmos.com/calculator/qrlbgbalix






                      share|cite|improve this answer











                      $endgroup$



                      A more generalised approach over my earlier post. This is not intended to be an exhaustive proof but an experimental one. Constructive comments are most welcome.



                      Let



                      $$sqrt{y^2-x}-x=Ay^nqquad cdots (1)\
                      sqrt{x^2+y}-y=frac {y^{1-n}}A qquad cdots (2)\$$
                      such that the original equation $$left(sqrt{y^2-x}-xright)left(sqrt{x^2+y}-yright)=y$$
                      is satisfied as required.



                      From $(1)$,



                      $$begin{align}
                      sqrt{y^2-x}&=x+Ay^n\
                      y^2-x&=x^2++2Axy^n+A^2y^{2n}\
                      y^{2n}A^2+2xy^nA+(x^2-y^2+x)&=0\
                      A^2+frac {2x}{y^n}A+frac{(x^2-y^2+x)}{y^{2n}}&=0qquad qquad qquad qquad cdots (3)
                      end{align}$$



                      From $(2)$,



                      $$begin{align}
                      sqrt{x^2+y}&=y+frac {y^{1-n}}A\
                      Asqrt{x^2+y}&=Ay+y^{1-n}\
                      A^2(x^2+y)&=A^2y^2+2Ay^{2-n}+y^{2(1-n)}\
                      (x^2-y^2+y)A^2-2y^{2-n}A-y^{2(1-n)}&=0\
                      A^2-frac{2y^{2-n}}{x^2-y^2+y}A-frac{y^{2(1-n)}}{x^2-y^2+y}&=0qquad qquad qquad cdots (4)
                      end{align}$$



                      Equating coefficients of $A^1$:
                      $$begin{align}frac{2x}{y^n}&=-frac{2y^{2-n}}{x^2-y^2+y}\
                      y^2&=-x(x^2-y^2+y)qquad qquad qquad qquad qquad qquad cdots (5)end{align}$$



                      Equating coefficients of $A^0$:



                      $$begin{align}frac{x^2-y^2+x}{y^{2n}}&=-frac{y^{2(1-n)}}{x^2-y^2+y}\
                      y^2&=-(x^2-y^2+x)(x^2-y^2+y)qquad cdots (6)end{align}$$



                      (5)=(6):
                      $$begin{align}x(x^2-y^2+y)&=(x^2-y^2+x)(x^2-y^2+y)\
                      (x^2-y^2)(x^2+y^2-y)&=0\
                      (x-y)(x+y)(x^2-y^2+y)&=0\
                      Rightarrow x-y=&0, x+y=0, x^2-y^2+y=0end{align}$$



                      Checking by substitution into the original equation shows that only
                      $$x+y=0$$
                      is valid.



                      This graph created on desmos.com might help illustrate the approach:



                      https://www.desmos.com/calculator/qrlbgbalix







                      share|cite|improve this answer














                      share|cite|improve this answer



                      share|cite|improve this answer








                      edited Aug 24 '14 at 6:06

























                      answered Aug 23 '14 at 5:58









                      hypergeometrichypergeometric

                      17.7k1758




                      17.7k1758












                      • $begingroup$
                        It's an interesting approach, but I don't buy it. Equations (3) and (4) show us that $A$ is a solution to the two indicated quadratic equations. It does not follow that they must be the same equation!
                        $endgroup$
                        – Bungo
                        Aug 23 '14 at 6:12










                      • $begingroup$
                        @Bungo Thanks for your comments. $A$ is actually a parameter in this case, and defines a series of curves for each equation. However at the same value of $A$ both equations intersect at a point which satisfies the original equation. The locus of this point is the solution required. Setting both quadratic equations to be the same is essentially assigning the same value of $A$ for both equations.
                        $endgroup$
                        – hypergeometric
                        Aug 23 '14 at 9:10










                      • $begingroup$
                        I agree that this is valid until you square the equations. But after squaring, each equation has a second root in addition to $A$. How do we know that the second root in (3) is the same as the second root in (4)? Sorry if I'm being dense. To take a simpler example: if $x = 1$ then for any $c$ I can express this as $x-c = 1-c$. Squaring both sides, I get $x^2 - 2xc + c^2 = 1 - 2c + c^2$, or $x^2 - 2xc - (1 - 2c) = 0$. If I carry out this procedure with two different values of $c$, I get two valid equations with $1$ as a root, but the coefficients are not the same.
                        $endgroup$
                        – Bungo
                        Aug 23 '14 at 16:40










                      • $begingroup$
                        That's true. By equating coefficients we arrive at one particular solution, but there may be others, e.g. where the two quadratics are not the same but have one common root...would it be correct to say that? Any suggestions on other possible approaches?
                        $endgroup$
                        – hypergeometric
                        Aug 23 '14 at 16:58










                      • $begingroup$
                        Yes, that's what I was getting at. If two quadratic equations have BOTH roots in common then their coefficients are the same, up to a common factor. (Actually that's one more issue with your approach: $x^2 + 2x + 1 = 0$ and $2x^2 + 4x + 2 = 0$ have the same roots but the coefficients are not equal.) Sorry, I don't have any suggestions at the moment - the other answer is pretty thorny and I haven't read through it yet. :-)
                        $endgroup$
                        – Bungo
                        Aug 23 '14 at 17:03


















                      • $begingroup$
                        It's an interesting approach, but I don't buy it. Equations (3) and (4) show us that $A$ is a solution to the two indicated quadratic equations. It does not follow that they must be the same equation!
                        $endgroup$
                        – Bungo
                        Aug 23 '14 at 6:12










                      • $begingroup$
                        @Bungo Thanks for your comments. $A$ is actually a parameter in this case, and defines a series of curves for each equation. However at the same value of $A$ both equations intersect at a point which satisfies the original equation. The locus of this point is the solution required. Setting both quadratic equations to be the same is essentially assigning the same value of $A$ for both equations.
                        $endgroup$
                        – hypergeometric
                        Aug 23 '14 at 9:10










                      • $begingroup$
                        I agree that this is valid until you square the equations. But after squaring, each equation has a second root in addition to $A$. How do we know that the second root in (3) is the same as the second root in (4)? Sorry if I'm being dense. To take a simpler example: if $x = 1$ then for any $c$ I can express this as $x-c = 1-c$. Squaring both sides, I get $x^2 - 2xc + c^2 = 1 - 2c + c^2$, or $x^2 - 2xc - (1 - 2c) = 0$. If I carry out this procedure with two different values of $c$, I get two valid equations with $1$ as a root, but the coefficients are not the same.
                        $endgroup$
                        – Bungo
                        Aug 23 '14 at 16:40










                      • $begingroup$
                        That's true. By equating coefficients we arrive at one particular solution, but there may be others, e.g. where the two quadratics are not the same but have one common root...would it be correct to say that? Any suggestions on other possible approaches?
                        $endgroup$
                        – hypergeometric
                        Aug 23 '14 at 16:58










                      • $begingroup$
                        Yes, that's what I was getting at. If two quadratic equations have BOTH roots in common then their coefficients are the same, up to a common factor. (Actually that's one more issue with your approach: $x^2 + 2x + 1 = 0$ and $2x^2 + 4x + 2 = 0$ have the same roots but the coefficients are not equal.) Sorry, I don't have any suggestions at the moment - the other answer is pretty thorny and I haven't read through it yet. :-)
                        $endgroup$
                        – Bungo
                        Aug 23 '14 at 17:03
















                      $begingroup$
                      It's an interesting approach, but I don't buy it. Equations (3) and (4) show us that $A$ is a solution to the two indicated quadratic equations. It does not follow that they must be the same equation!
                      $endgroup$
                      – Bungo
                      Aug 23 '14 at 6:12




                      $begingroup$
                      It's an interesting approach, but I don't buy it. Equations (3) and (4) show us that $A$ is a solution to the two indicated quadratic equations. It does not follow that they must be the same equation!
                      $endgroup$
                      – Bungo
                      Aug 23 '14 at 6:12












                      $begingroup$
                      @Bungo Thanks for your comments. $A$ is actually a parameter in this case, and defines a series of curves for each equation. However at the same value of $A$ both equations intersect at a point which satisfies the original equation. The locus of this point is the solution required. Setting both quadratic equations to be the same is essentially assigning the same value of $A$ for both equations.
                      $endgroup$
                      – hypergeometric
                      Aug 23 '14 at 9:10




                      $begingroup$
                      @Bungo Thanks for your comments. $A$ is actually a parameter in this case, and defines a series of curves for each equation. However at the same value of $A$ both equations intersect at a point which satisfies the original equation. The locus of this point is the solution required. Setting both quadratic equations to be the same is essentially assigning the same value of $A$ for both equations.
                      $endgroup$
                      – hypergeometric
                      Aug 23 '14 at 9:10












                      $begingroup$
                      I agree that this is valid until you square the equations. But after squaring, each equation has a second root in addition to $A$. How do we know that the second root in (3) is the same as the second root in (4)? Sorry if I'm being dense. To take a simpler example: if $x = 1$ then for any $c$ I can express this as $x-c = 1-c$. Squaring both sides, I get $x^2 - 2xc + c^2 = 1 - 2c + c^2$, or $x^2 - 2xc - (1 - 2c) = 0$. If I carry out this procedure with two different values of $c$, I get two valid equations with $1$ as a root, but the coefficients are not the same.
                      $endgroup$
                      – Bungo
                      Aug 23 '14 at 16:40




                      $begingroup$
                      I agree that this is valid until you square the equations. But after squaring, each equation has a second root in addition to $A$. How do we know that the second root in (3) is the same as the second root in (4)? Sorry if I'm being dense. To take a simpler example: if $x = 1$ then for any $c$ I can express this as $x-c = 1-c$. Squaring both sides, I get $x^2 - 2xc + c^2 = 1 - 2c + c^2$, or $x^2 - 2xc - (1 - 2c) = 0$. If I carry out this procedure with two different values of $c$, I get two valid equations with $1$ as a root, but the coefficients are not the same.
                      $endgroup$
                      – Bungo
                      Aug 23 '14 at 16:40












                      $begingroup$
                      That's true. By equating coefficients we arrive at one particular solution, but there may be others, e.g. where the two quadratics are not the same but have one common root...would it be correct to say that? Any suggestions on other possible approaches?
                      $endgroup$
                      – hypergeometric
                      Aug 23 '14 at 16:58




                      $begingroup$
                      That's true. By equating coefficients we arrive at one particular solution, but there may be others, e.g. where the two quadratics are not the same but have one common root...would it be correct to say that? Any suggestions on other possible approaches?
                      $endgroup$
                      – hypergeometric
                      Aug 23 '14 at 16:58












                      $begingroup$
                      Yes, that's what I was getting at. If two quadratic equations have BOTH roots in common then their coefficients are the same, up to a common factor. (Actually that's one more issue with your approach: $x^2 + 2x + 1 = 0$ and $2x^2 + 4x + 2 = 0$ have the same roots but the coefficients are not equal.) Sorry, I don't have any suggestions at the moment - the other answer is pretty thorny and I haven't read through it yet. :-)
                      $endgroup$
                      – Bungo
                      Aug 23 '14 at 17:03




                      $begingroup$
                      Yes, that's what I was getting at. If two quadratic equations have BOTH roots in common then their coefficients are the same, up to a common factor. (Actually that's one more issue with your approach: $x^2 + 2x + 1 = 0$ and $2x^2 + 4x + 2 = 0$ have the same roots but the coefficients are not equal.) Sorry, I don't have any suggestions at the moment - the other answer is pretty thorny and I haven't read through it yet. :-)
                      $endgroup$
                      – Bungo
                      Aug 23 '14 at 17:03











                      0












                      $begingroup$

                      We need to prove that $x=y$, where
                      $$left(sqrt{y^{2}+x}+xright)left(sqrt{x^{2} + y}- yright)=y$$ or



                      $$left(sqrt{y^{2}+x}+xright)left(sqrt{x^{2} + y}- yright)=left(sqrt{x^{2}+y}+xright)left(sqrt{x^{2} + y}- xright)$$ or
                      $$left(sqrt{y^{2}+x}+xright)left(sqrt{x^{2} + y}- yright)-left(sqrt{x^{2}+y}+xright)left(sqrt{x^{2} + y}- yright)+$$
                      $$+left(sqrt{x^{2}+y}+xright)left(sqrt{x^{2} + y}- yright)-left(sqrt{x^{2}+y}+xright)left(sqrt{x^{2} + y}- xright)=0$$ or
                      $$left(sqrt{y^{2}+x}-sqrt{x^2+y}right)left(sqrt{x^{2} + y}- yright)+left(sqrt{x^{2}+y}+xright)(x-y)=0,$$ which gives $x=y$ or



                      $$frac{(1-x-y)left(sqrt{x^{2} + y}- yright)}{sqrt{y^{2}+x}+sqrt{x^2+y}}+sqrt{x^{2}+y}+x=0,$$ which is
                      $$sqrt{x^2+y}-ysqrt{x^2+y}+sqrt{(x^2+y)(y^2+x)}+xsqrt{y^2+x}+x^2+xy+y^2=0.$$
                      Now, we'll consider four cases.





                      1. $xgeq0$, $ygeq 0$.


                      Since $$-ysqrt{x^2+y}+sqrt{(x^2+y)(y^2+x)}=sqrt{x^2+y}left(sqrt{y^2+x}-yright)geq0,$$ we obtain $x=y=0.$





                      1. $xgeq0,$ $yleq0.$


                      It's obvious that this case gives $x=y=0$ again.





                      1. $xleq0$, $ygeq0.$


                      Since, $$sqrt{(x^2+y)(y^2+x)}+xsqrt{y^2+x}=sqrt{y^2+x}left(sqrt{x^2+y}+xright)geq0,$$ it's enough to prove that
                      $$x^2+xy+y^2geq(y-1)sqrt{x^2+y},$$ which is obvious for $yleq1.$



                      But for $ygeq1$ by AM-GM we obtain:
                      $$(y-1)sqrt{x^2+y}leqfrac{1}{2}((y-1)^2+x^2+y)$$ and it's enough to prove that
                      $$x^2+xy+y^2geqfrac{1}{2}((y-1)^2+x^2+y)$$ or
                      $$require{cancel} cancel{(x+y)^2+y^2+y-1geq0.}\
                      (x+y)^2+y-1geq0.$$

                      We see that for $ygeq1$ the equality does not occur and in the case $y<1$ the equality occurs for
                      $$x^2+xy+y^2=(y-1)sqrt{x^2+y}=0,$$ which gives $x=y=0$ again.





                      1. $xleq0$ and $yleq0.$


                      In this case it's enough to prove that
                      $$xy+xsqrt{y^2+x}geq0$$ or
                      $$xleft(y+sqrt{y^2+x}right)geq0,$$ which is obvious.



                      The equality occurs for $x^2+y^2=0$ and we got $x=y=0$ again.



                      Done!






                      share|cite|improve this answer











                      $endgroup$













                      • $begingroup$
                        Very nice proof (+1)!
                        $endgroup$
                        – Andreas
                        Dec 5 '18 at 15:38










                      • $begingroup$
                        @Andreas Thank you and thank you for your editing. Do you see that my solution it's an unique right solution in this topic?
                        $endgroup$
                        – Michael Rozenberg
                        Dec 5 '18 at 15:53










                      • $begingroup$
                        Your solution is the only valid one in here, as far as I see it, which goes without calculus. There may be other approaches with calculus, I didn't check that fully.
                        $endgroup$
                        – Andreas
                        Dec 6 '18 at 7:23
















                      0












                      $begingroup$

                      We need to prove that $x=y$, where
                      $$left(sqrt{y^{2}+x}+xright)left(sqrt{x^{2} + y}- yright)=y$$ or



                      $$left(sqrt{y^{2}+x}+xright)left(sqrt{x^{2} + y}- yright)=left(sqrt{x^{2}+y}+xright)left(sqrt{x^{2} + y}- xright)$$ or
                      $$left(sqrt{y^{2}+x}+xright)left(sqrt{x^{2} + y}- yright)-left(sqrt{x^{2}+y}+xright)left(sqrt{x^{2} + y}- yright)+$$
                      $$+left(sqrt{x^{2}+y}+xright)left(sqrt{x^{2} + y}- yright)-left(sqrt{x^{2}+y}+xright)left(sqrt{x^{2} + y}- xright)=0$$ or
                      $$left(sqrt{y^{2}+x}-sqrt{x^2+y}right)left(sqrt{x^{2} + y}- yright)+left(sqrt{x^{2}+y}+xright)(x-y)=0,$$ which gives $x=y$ or



                      $$frac{(1-x-y)left(sqrt{x^{2} + y}- yright)}{sqrt{y^{2}+x}+sqrt{x^2+y}}+sqrt{x^{2}+y}+x=0,$$ which is
                      $$sqrt{x^2+y}-ysqrt{x^2+y}+sqrt{(x^2+y)(y^2+x)}+xsqrt{y^2+x}+x^2+xy+y^2=0.$$
                      Now, we'll consider four cases.





                      1. $xgeq0$, $ygeq 0$.


                      Since $$-ysqrt{x^2+y}+sqrt{(x^2+y)(y^2+x)}=sqrt{x^2+y}left(sqrt{y^2+x}-yright)geq0,$$ we obtain $x=y=0.$





                      1. $xgeq0,$ $yleq0.$


                      It's obvious that this case gives $x=y=0$ again.





                      1. $xleq0$, $ygeq0.$


                      Since, $$sqrt{(x^2+y)(y^2+x)}+xsqrt{y^2+x}=sqrt{y^2+x}left(sqrt{x^2+y}+xright)geq0,$$ it's enough to prove that
                      $$x^2+xy+y^2geq(y-1)sqrt{x^2+y},$$ which is obvious for $yleq1.$



                      But for $ygeq1$ by AM-GM we obtain:
                      $$(y-1)sqrt{x^2+y}leqfrac{1}{2}((y-1)^2+x^2+y)$$ and it's enough to prove that
                      $$x^2+xy+y^2geqfrac{1}{2}((y-1)^2+x^2+y)$$ or
                      $$require{cancel} cancel{(x+y)^2+y^2+y-1geq0.}\
                      (x+y)^2+y-1geq0.$$

                      We see that for $ygeq1$ the equality does not occur and in the case $y<1$ the equality occurs for
                      $$x^2+xy+y^2=(y-1)sqrt{x^2+y}=0,$$ which gives $x=y=0$ again.





                      1. $xleq0$ and $yleq0.$


                      In this case it's enough to prove that
                      $$xy+xsqrt{y^2+x}geq0$$ or
                      $$xleft(y+sqrt{y^2+x}right)geq0,$$ which is obvious.



                      The equality occurs for $x^2+y^2=0$ and we got $x=y=0$ again.



                      Done!






                      share|cite|improve this answer











                      $endgroup$













                      • $begingroup$
                        Very nice proof (+1)!
                        $endgroup$
                        – Andreas
                        Dec 5 '18 at 15:38










                      • $begingroup$
                        @Andreas Thank you and thank you for your editing. Do you see that my solution it's an unique right solution in this topic?
                        $endgroup$
                        – Michael Rozenberg
                        Dec 5 '18 at 15:53










                      • $begingroup$
                        Your solution is the only valid one in here, as far as I see it, which goes without calculus. There may be other approaches with calculus, I didn't check that fully.
                        $endgroup$
                        – Andreas
                        Dec 6 '18 at 7:23














                      0












                      0








                      0





                      $begingroup$

                      We need to prove that $x=y$, where
                      $$left(sqrt{y^{2}+x}+xright)left(sqrt{x^{2} + y}- yright)=y$$ or



                      $$left(sqrt{y^{2}+x}+xright)left(sqrt{x^{2} + y}- yright)=left(sqrt{x^{2}+y}+xright)left(sqrt{x^{2} + y}- xright)$$ or
                      $$left(sqrt{y^{2}+x}+xright)left(sqrt{x^{2} + y}- yright)-left(sqrt{x^{2}+y}+xright)left(sqrt{x^{2} + y}- yright)+$$
                      $$+left(sqrt{x^{2}+y}+xright)left(sqrt{x^{2} + y}- yright)-left(sqrt{x^{2}+y}+xright)left(sqrt{x^{2} + y}- xright)=0$$ or
                      $$left(sqrt{y^{2}+x}-sqrt{x^2+y}right)left(sqrt{x^{2} + y}- yright)+left(sqrt{x^{2}+y}+xright)(x-y)=0,$$ which gives $x=y$ or



                      $$frac{(1-x-y)left(sqrt{x^{2} + y}- yright)}{sqrt{y^{2}+x}+sqrt{x^2+y}}+sqrt{x^{2}+y}+x=0,$$ which is
                      $$sqrt{x^2+y}-ysqrt{x^2+y}+sqrt{(x^2+y)(y^2+x)}+xsqrt{y^2+x}+x^2+xy+y^2=0.$$
                      Now, we'll consider four cases.





                      1. $xgeq0$, $ygeq 0$.


                      Since $$-ysqrt{x^2+y}+sqrt{(x^2+y)(y^2+x)}=sqrt{x^2+y}left(sqrt{y^2+x}-yright)geq0,$$ we obtain $x=y=0.$





                      1. $xgeq0,$ $yleq0.$


                      It's obvious that this case gives $x=y=0$ again.





                      1. $xleq0$, $ygeq0.$


                      Since, $$sqrt{(x^2+y)(y^2+x)}+xsqrt{y^2+x}=sqrt{y^2+x}left(sqrt{x^2+y}+xright)geq0,$$ it's enough to prove that
                      $$x^2+xy+y^2geq(y-1)sqrt{x^2+y},$$ which is obvious for $yleq1.$



                      But for $ygeq1$ by AM-GM we obtain:
                      $$(y-1)sqrt{x^2+y}leqfrac{1}{2}((y-1)^2+x^2+y)$$ and it's enough to prove that
                      $$x^2+xy+y^2geqfrac{1}{2}((y-1)^2+x^2+y)$$ or
                      $$require{cancel} cancel{(x+y)^2+y^2+y-1geq0.}\
                      (x+y)^2+y-1geq0.$$

                      We see that for $ygeq1$ the equality does not occur and in the case $y<1$ the equality occurs for
                      $$x^2+xy+y^2=(y-1)sqrt{x^2+y}=0,$$ which gives $x=y=0$ again.





                      1. $xleq0$ and $yleq0.$


                      In this case it's enough to prove that
                      $$xy+xsqrt{y^2+x}geq0$$ or
                      $$xleft(y+sqrt{y^2+x}right)geq0,$$ which is obvious.



                      The equality occurs for $x^2+y^2=0$ and we got $x=y=0$ again.



                      Done!






                      share|cite|improve this answer











                      $endgroup$



                      We need to prove that $x=y$, where
                      $$left(sqrt{y^{2}+x}+xright)left(sqrt{x^{2} + y}- yright)=y$$ or



                      $$left(sqrt{y^{2}+x}+xright)left(sqrt{x^{2} + y}- yright)=left(sqrt{x^{2}+y}+xright)left(sqrt{x^{2} + y}- xright)$$ or
                      $$left(sqrt{y^{2}+x}+xright)left(sqrt{x^{2} + y}- yright)-left(sqrt{x^{2}+y}+xright)left(sqrt{x^{2} + y}- yright)+$$
                      $$+left(sqrt{x^{2}+y}+xright)left(sqrt{x^{2} + y}- yright)-left(sqrt{x^{2}+y}+xright)left(sqrt{x^{2} + y}- xright)=0$$ or
                      $$left(sqrt{y^{2}+x}-sqrt{x^2+y}right)left(sqrt{x^{2} + y}- yright)+left(sqrt{x^{2}+y}+xright)(x-y)=0,$$ which gives $x=y$ or



                      $$frac{(1-x-y)left(sqrt{x^{2} + y}- yright)}{sqrt{y^{2}+x}+sqrt{x^2+y}}+sqrt{x^{2}+y}+x=0,$$ which is
                      $$sqrt{x^2+y}-ysqrt{x^2+y}+sqrt{(x^2+y)(y^2+x)}+xsqrt{y^2+x}+x^2+xy+y^2=0.$$
                      Now, we'll consider four cases.





                      1. $xgeq0$, $ygeq 0$.


                      Since $$-ysqrt{x^2+y}+sqrt{(x^2+y)(y^2+x)}=sqrt{x^2+y}left(sqrt{y^2+x}-yright)geq0,$$ we obtain $x=y=0.$





                      1. $xgeq0,$ $yleq0.$


                      It's obvious that this case gives $x=y=0$ again.





                      1. $xleq0$, $ygeq0.$


                      Since, $$sqrt{(x^2+y)(y^2+x)}+xsqrt{y^2+x}=sqrt{y^2+x}left(sqrt{x^2+y}+xright)geq0,$$ it's enough to prove that
                      $$x^2+xy+y^2geq(y-1)sqrt{x^2+y},$$ which is obvious for $yleq1.$



                      But for $ygeq1$ by AM-GM we obtain:
                      $$(y-1)sqrt{x^2+y}leqfrac{1}{2}((y-1)^2+x^2+y)$$ and it's enough to prove that
                      $$x^2+xy+y^2geqfrac{1}{2}((y-1)^2+x^2+y)$$ or
                      $$require{cancel} cancel{(x+y)^2+y^2+y-1geq0.}\
                      (x+y)^2+y-1geq0.$$

                      We see that for $ygeq1$ the equality does not occur and in the case $y<1$ the equality occurs for
                      $$x^2+xy+y^2=(y-1)sqrt{x^2+y}=0,$$ which gives $x=y=0$ again.





                      1. $xleq0$ and $yleq0.$


                      In this case it's enough to prove that
                      $$xy+xsqrt{y^2+x}geq0$$ or
                      $$xleft(y+sqrt{y^2+x}right)geq0,$$ which is obvious.



                      The equality occurs for $x^2+y^2=0$ and we got $x=y=0$ again.



                      Done!







                      share|cite|improve this answer














                      share|cite|improve this answer



                      share|cite|improve this answer








                      edited Dec 5 '18 at 17:49

























                      answered Nov 26 '18 at 12:50









                      Michael RozenbergMichael Rozenberg

                      99.1k1590189




                      99.1k1590189












                      • $begingroup$
                        Very nice proof (+1)!
                        $endgroup$
                        – Andreas
                        Dec 5 '18 at 15:38










                      • $begingroup$
                        @Andreas Thank you and thank you for your editing. Do you see that my solution it's an unique right solution in this topic?
                        $endgroup$
                        – Michael Rozenberg
                        Dec 5 '18 at 15:53










                      • $begingroup$
                        Your solution is the only valid one in here, as far as I see it, which goes without calculus. There may be other approaches with calculus, I didn't check that fully.
                        $endgroup$
                        – Andreas
                        Dec 6 '18 at 7:23


















                      • $begingroup$
                        Very nice proof (+1)!
                        $endgroup$
                        – Andreas
                        Dec 5 '18 at 15:38










                      • $begingroup$
                        @Andreas Thank you and thank you for your editing. Do you see that my solution it's an unique right solution in this topic?
                        $endgroup$
                        – Michael Rozenberg
                        Dec 5 '18 at 15:53










                      • $begingroup$
                        Your solution is the only valid one in here, as far as I see it, which goes without calculus. There may be other approaches with calculus, I didn't check that fully.
                        $endgroup$
                        – Andreas
                        Dec 6 '18 at 7:23
















                      $begingroup$
                      Very nice proof (+1)!
                      $endgroup$
                      – Andreas
                      Dec 5 '18 at 15:38




                      $begingroup$
                      Very nice proof (+1)!
                      $endgroup$
                      – Andreas
                      Dec 5 '18 at 15:38












                      $begingroup$
                      @Andreas Thank you and thank you for your editing. Do you see that my solution it's an unique right solution in this topic?
                      $endgroup$
                      – Michael Rozenberg
                      Dec 5 '18 at 15:53




                      $begingroup$
                      @Andreas Thank you and thank you for your editing. Do you see that my solution it's an unique right solution in this topic?
                      $endgroup$
                      – Michael Rozenberg
                      Dec 5 '18 at 15:53












                      $begingroup$
                      Your solution is the only valid one in here, as far as I see it, which goes without calculus. There may be other approaches with calculus, I didn't check that fully.
                      $endgroup$
                      – Andreas
                      Dec 6 '18 at 7:23




                      $begingroup$
                      Your solution is the only valid one in here, as far as I see it, which goes without calculus. There may be other approaches with calculus, I didn't check that fully.
                      $endgroup$
                      – Andreas
                      Dec 6 '18 at 7:23


















                      draft saved

                      draft discarded




















































                      Thanks for contributing an answer to Mathematics Stack Exchange!


                      • Please be sure to answer the question. Provide details and share your research!

                      But avoid



                      • Asking for help, clarification, or responding to other answers.

                      • Making statements based on opinion; back them up with references or personal experience.


                      Use MathJax to format equations. MathJax reference.


                      To learn more, see our tips on writing great answers.




                      draft saved


                      draft discarded














                      StackExchange.ready(
                      function () {
                      StackExchange.openid.initPostLogin('.new-post-login', 'https%3a%2f%2fmath.stackexchange.com%2fquestions%2f543607%2fshow-that-sqrty2-x-x-sqrtx2y-y-y-iff-xy-0%23new-answer', 'question_page');
                      }
                      );

                      Post as a guest















                      Required, but never shown





















































                      Required, but never shown














                      Required, but never shown












                      Required, but never shown







                      Required, but never shown

































                      Required, but never shown














                      Required, but never shown












                      Required, but never shown







                      Required, but never shown







                      Popular posts from this blog

                      How do I know what Microsoft account the skydrive app is syncing to?

                      When does type information flow backwards in C++?

                      Grease: Live!